[obm-l] Re: [obm-l] n-ésima derivada de arc sen x

2004-08-01 Por tôpico Marcio Afonso A. Cohen
Para isso, eh suficiente conhecer a expansao de Taylor do arcsenx em
torno do ponto x=0.
a)Note que arcsenx eh a integral indefinida de (1-x^2)^(-1/2), cuja
série voce obtem pelo binomio de Newton.
b)Sendo u = arcsen x, y=u^2 implica y' = 2uu', e em geral, a derivada
n-esima de y'/2 eh dada por
Somatorio_k=0..n_ Binomial(n,k)*u^(k)*u^(n-k+1), onde u^l eh a derivada
l-esima de u, que voce ja sabe quanto vale em x=0 pela letra a. Pronto. Com
um pouco de paciencia voce completa os detalhes e tem a resposta para as
duas perguntas.
[]s
Marcio

- Original Message -
From: [EMAIL PROTECTED]
To: [EMAIL PROTECTED]
Sent: Sunday, August 01, 2004 6:23 PM
Subject: [obm-l] n-ésima derivada de arc sen x


 Uma ajuda no problema abaixo:

 Calcule a n-ésima derivada de arc sen x no ponto x=0. Calcule também a n-
 ésima derivada de (arc sen x)^2 em x=0.

 []s,
 Daniel

 =
 Instruções para entrar na lista, sair da lista e usar a lista em
 http://www.mat.puc-rio.br/~nicolau/olimp/obm-l.html
 =


=
Instruções para entrar na lista, sair da lista e usar a lista em
http://www.mat.puc-rio.br/~nicolau/olimp/obm-l.html
=


Re: [obm-l] Resultado da IMC

2004-07-28 Por tôpico Marcio Afonso A. Cohen
Meus parabens ao pessoal do Brasil!! Em especial para o Yuri, pelo
excelente resultado, e para Alex, Stein e Humberto, que embora tenham ficado
com a prata pelo jeito mereciam o ouro!
Abraços a todos!
Marcio


- Original Message -
From: alex.abreu [EMAIL PROTECTED]
To: obm-l [EMAIL PROTECTED]
Sent: Wednesday, July 28, 2004 4:36 PM
Subject: [obm-l] Resultado da IMC


 Ola a todos da lista,

  Segue abaixo o resultado da IMC - 2004. Tivemos muito azar
 com os cortes. Foram eles:

  OURO - 131PRATA - 108 BRONZE - 73

 NOME1  2  3  4  5  6  7  8  9  10  11  12 TOTAL  PREMIO
 Alex20 0  14 20 0  0  20 20 2  15  0   0   111  2nd Prize

 Carlos  20 5  12 18 19 0  20 3  17 0   0   0   114  2nd Prize
 Stein

 Diego   19 20 7  0  0  0  20 0  2  0   0   0   68Mencao

 Eduardo 13 20 0  18 0  0  2  0  0  0   0   0   53Mencao
 Famini

 Eduardo 20 20 15 0  0  0  20 0  20 0   0   0   95  3rd Prize
 Casagrande

 Humberto 19 12 14 0 5  4  20 20 20 16  0   0   130  2nd Prize

 Murilo  20 14 10 18 0  0  20 0  0  1   1   0   84  3rd Prize

 Rafael  20 20 8  0  0  0  20 2  8  0   0   0   78  3rd Prize

 Tertuliano
 8  20 2  5  0  0  0  3  0  0   0   0   38Mencao

 Thiago  20 12 0  20 0  0  20 0  0  0   0   0   72Mencao
 Barros

 Yuri20 20 20 20 0  0  20 0  18 10  20  0   148  1st Prize


 Vejam que tivemos a primeira prata e a primeira mencao. Alem
 disso, tivemos problemas com a correcao (muitos).

  Abracos,

  Alex

 __
 Acabe com aquelas janelinhas que pulam na sua tela.
 AntiPop-up UOL - É grátis!
 http://antipopup.uol.com.br/



 =
 Instruções para entrar na lista, sair da lista e usar a lista em
 http://www.mat.puc-rio.br/~nicolau/olimp/obm-l.html
 =


=
Instruções para entrar na lista, sair da lista e usar a lista em
http://www.mat.puc-rio.br/~nicolau/olimp/obm-l.html
=


[obm-l] IMC, 1o dia: Solucoes 1,2,3,6.

2004-07-26 Por tôpico Marcio Afonso A. Cohen



 Oi 
gente.. Tentei fazer as questoes do 1o dia da imc. Estamos ansiosospor 
noticias de como o pessoal esta indo na prova! Ao contrario da IMO, naIMC 
nao eh o lider que corrige as provas do pais. Ele participa da banca deuma 
determinada questao e depois participa da revisao de notas dos seusalunos. 
Por esse motivo, seria interessante que voces postassem asprincipais 
solucoes para que o pessoal da lista possa ajudar.. Quem 
nao quiser ver as solucoes, pare de ler por aqui. Vou escreverminhas 
solucoes, leiam com atencao e me avisem se notarem algoerrado. O 1,2,3 eram 
mais faceis. O 5 eu escrevi 2 linhas sem mto nexo la embaixo :) e no 6 eu 
usei um resultado nao obvio de analise complexa (e puleialgumas contas que 
teriam de ser feitas na prova).Ja o 4 eu demorei bastante (um pouco mais q o 
2o tempo inteiro do jogo BrasilxArgentina) e achei que tinha feito. Mas depois 
eu fui ver a solucao do mathlinks e vi que era bem curta, mto simples, e comeco 
a achar que a minha esta errada (alem de ser completamente gigante). 

 Em tempo: eu acabei de ver a 
prova do 2o dia. A primeira eh bem simples, usa uma ideia analoga a que foi 
usada na 1 do 2o dia do ano passado :) Mas as outras parecem estar bem mais 
dificeis do que as de hoje!! Nao tenho ideia alguma para a 2 por exemplo.. 

 Abracos,
 Marcio 1) Let S 
be an infinite set of real numbers such that |s_1 + s_2 + ... + s_k| 
for every finite subset {s_1,s_2,...,s_k} of S. Show that S is 
countable Escreva S = A U B, onde A e B sao 
respectivamente os subconjuntos denao-negativos e nao-positivos de 
S. Vamos provar que A e B sao enumeraveis e portanto S tmb 
é (por ser uniaode 2 enumeraveis). A enumeracao eh a 
seguinte (A e B sao analogos). Se A eh finito, ehsimples. Caso 
contrario,coloque x_1 = max{A}, x_2 = max{A-{x_1}}, x_3 =max{A-{x_1,x_2}}, e 
assim por diante. Isso funciona pq todos esses conjuntos 
admitem maximo!De fato, suponhaque um conjunto X com infinitos termos e tq 
todo subconjunto tem modulo dasoma de seus elementos maior que 1 nao tenha 
maximo. Seja c = supX. Entao,existem infinitos elementos de X no intervalo 
(c/2, c) (pq em todointervalo (c-eps, c) deve haver elemento de X e vc 
sempre pega um 'maisproximo' de c) e pegando [2/c]+1 deles, a soma eh maior 
que (2/c)*c/2 = 1. 2)Let P(x) = x^2 - 1. How many distinct real 
solutions does the following equation have: 
P(P(...(P(x))...)) = 0? [com P sendo aplicado 2004 
vezes] Tem 2005 raizes. Vamos provar dois resultados 
por inducao que claramenteimplicam isso. (Vou chamar P(...(P(x)...) 
com P aplicado k vezes de P_k(x))(i) A equacao P_n (x) = 0 tem sempre 
n+1 solucoes.(ii)A equacao [P_n (x) ]^2 = 1+a, com a0 tem sempre 2 
solucoes. Para n = 1, o resultado eh bem obvio, basta 
olhar pro grafico de p(x) =x^2 - 1. Para n = 2, tmb eh 
bem direto.Suponha valido ateh n. (i) Considere a 
equacao P_n+1 (x) = 1+a, a0 equivale a ([P_n (x)]^2 -1)^2 = 1+a. Como 
P_n ^2 eh sempre positivo, isso eh equivalente a (P_n(x))^2 = 1 + sqrt(a), 
que por hipotese de inducao tem 2 solucoes. (ii) P_n+1 (x) 
= 0 sse (P_n (x))^2 - 1 = 0 sse P_n(x) = 1 ou P_n(x)= -1. No primeiro caso, 
[P_n-1 (x)]^2 - 1 = 1, e pela hipotese de inducao(ii) isso tem exatamente 
duas raizes (equivale a (P_n-1 (x) )^2 = sqrt(2) 1). No segundo caso, 
[P_n-1 (x)]^2 - 1 = -1 sse P_n-1(x) = 0 e pela hipotesede inducao (i) isso 
tem exatamente n-1+1 = n raizes (distintas das outras2), de forma que a 
equacaoP_n+1 (x) = 0 tem n+2 solucoes de fato. 3) Let S_n be 
the set of all sum x_1+x_2+...x_n, where n=2, 
0=x_1,...,x_n="pi"/2 and sin(x_1) + sin(x_2) + ... + 
sin(x_n) = 1 a) Show that S_n is an interval. b)Let l_n 
be the length of S_n. Find lim(n-infinito)(l_n).a) Por Jensen, 
sen[(x_1+...x_n)/n] = (senx1+ ... +sen xn)/n = 1/n. Logo,x1+...+xn = 
n * arcsen(1/n). Sabemos que senx = 2x/pi para x em 
[0,pi/2] (isso segue da concavidadedo seno), de forma quex1+x2+...+xn 
= pi/2 (senx1+...senxn) = pi/2. Para mostrar que a imagem eh 
o intervalo (n * arcsen(1/n), pi/2) vocepode soh citar a continuidade da 
funcao ou mostrar usando soh o TVIunidimensional, vendo que dado t nesse 
intervalo, sempre consigo escolher ade forma que x1=x2=...=x_n-1 = a/(n-1), 
x_n = t -a e (n-1)sen[a/(n-1)] +sen(t-a) = 1 (encare o lhs como um 
f(a) e veja que f(0) = sen t = 1 e sendoa' tq a'/(n-1) = t-a', f(a') = 
n*arcsen(1/n) = 1).b) A letra (b) eh consequencia do limite fundamental 
senx/x = 1 qdo x - 0.A resposta eh portanto pi/2 - 1. 
4)Suppose n=4 and let M be a finite set of n points in R^3, 
no four of which lie in a plane. Assume that the points can be 
coloured black or white so that any of the sphere which intersect M 
in at least four points have the property that exactly half of the 
points in the intersection of M and the sphere are white. 
Prove that all of the points in M lie on one sphere.  ... 
 ... 5) Let X be a set of 
binomial(2k-4, k-2) + 1 real numbers, k=2. Prove that 
there exists a monotone sequence x_1, x_2, ..., x_k in X 

Re: [obm-l] polinomio homogeneo

2004-07-02 Por tôpico Marcio Afonso A. Cohen
Claro que não. Pegue um exemplo qualquer tipo f(x,y,z) = -x^2  e
provavelmente voce já vai se dar conta de que nao tem relacao nenhuma.

- Original Message -
From: leonardo mattos [EMAIL PROTECTED]
To: [EMAIL PROTECTED]
Sent: Friday, July 02, 2004 11:01 PM
Subject: [obm-l] polinomio homogeneo


 Ola,

 Seja f(x,y,z) tal que f(tx,ty,tz)=t²*f(x,y,z), ou seja f eh homogenea de
 grau 2. Isso implica em
 f(x,y,z)=0 para t0?

 Um abraço,
 Leonardo

 _
 MSN Hotmail, o maior webmail do Brasil.  http://www.hotmail.com

 =
 Instruções para entrar na lista, sair da lista e usar a lista em
 http://www.mat.puc-rio.br/~nicolau/olimp/obm-l.html
 =


=
Instruções para entrar na lista, sair da lista e usar a lista em
http://www.mat.puc-rio.br/~nicolau/olimp/obm-l.html
=


[obm-l] Re: [obm-l] Re: [obm-l] Área máxima

2004-07-02 Por tôpico Marcio Afonso A. Cohen
Não consigo dizer se voce está certo ou errado. A conclusão está correta
(a resposta é de fato o equilátero), mas eu pelo menos não consegui enxergar
nenhuma ligação direta entre o fato de a área ser A = abc/4R e o seu máximo
ser atingido no equilátero.. Por que o fato de se ter A = abc / 4r implica
que A é máxima quando a=b=c???
Segue abaixo uma solucao para o caso geral.
Dado um poligono convexo de n lados, chame de x_1, x_2, ..., x_n os angulos
centrais que enxergam como cordas os lados do poligono. A area de cada
triangulo formado por um lado e o centro da circunferencia eh
(1/2)*(R^2)sen(x_k), de modo que a área total é:
S = (1/2)*(R^2)*[sen(x_1)+sen(x_2)+...sen(x_n)]
Como f(x) = senx tem segunda derivada f''(x)  0 em (0,pi) e todos os
x_i estao nesse intervalo, a desigualdade de Jensen nos dá:
[sen(x_1)+sen(x_2)+...+sen(x_n)]/n = sen[(x_1+x_2+...+x_n)/n] =
sen(2pi/n), com igualdade sse x_1=x_2=...=x_n (e portanto o polígono é
regular).
Portanto, S = (1/2)*R^2*n*sen(2pi/n), com igualdade sse o polígono é
regular.
Isso responde a pergunta original, que perguntava como deve ser um
polígono inscrito de área máxima.
[]s
Marcio



- Original Message -
From: [EMAIL PROTECTED]
To: [EMAIL PROTECTED]
Sent: Friday, July 02, 2004 10:26 PM
Subject: [obm-l] Re: [obm-l] Área máxima




  Caro, Igor

  Considere um triangulo de lados a , b e c , inscrito numa circunferencia
de
 raio r , a área desse triangulo em funçao de r eh dada por A=a.b.c/4r ,
logo o
 triangulo terá área máxima quando a=b=c( Equilátero)se estou errado me
corrijam
 por favor.( Igor se vv não souber de onde vem está fórmula da área me
escreva
 que mandarei a desmostraçao para vc ok! ( [EMAIL PROTECTED] )

  Espero ter ajudado.

   Cláudio Thor





 Citando Igor Oliveira [EMAIL PROTECTED]:

  Gostaria de saber como faço pra achar o triângulo de área máxima
inscrito
  numa
  circunferência. É o eqüilátero? E o polígono de n lados com área máxima
e
  inscrito
  numa é sempre o polígono regular de n lados? Obrigado
 
  Igor
 
=
  Instruções para entrar na lista, sair da lista e usar a lista em
  http://www.mat.puc-rio.br/~nicolau/olimp/obm-l.html
 
=
 



 =
 Instruções para entrar na lista, sair da lista e usar a lista em
 http://www.mat.puc-rio.br/~nicolau/olimp/obm-l.html
 =


=
Instruções para entrar na lista, sair da lista e usar a lista em
http://www.mat.puc-rio.br/~nicolau/olimp/obm-l.html
=


Re: [obm-l] OBM - 1997

2004-06-27 Por tôpico Marcio Afonso A. Cohen



 É bem simples formalizar que 
essa equação só tem uma raiz negativa (2^x - x^2 eh crescente e vem de -oo até 
1), mas nao eh tao obvio assim a formalizacao de que no total só temos tres 
raizes. Uma maneira de formalizar eh a q se segue:
 Seja f(x) = 2^x - x^2. 
Como f(-1) = 1/2 - 1  0 e f(0) = 1  0, f tem ao menos uma raiz em 
(-1,0). Alem disso, f(2)=f(4)=0, logo f tem pelo menos 3 raizes.
 Suponha por absurdo que f tenha 
pelo menos 4 raizes reais x1,x2,x3,x4. Pelo teorema de Rolle, f'(x) tem ao menos 
3 raizes reais (em (x1,x2),(x2,x3),(x3,x4)). Analogamente, o teorema de Rolle 
garante que f" tem ao menos 2 raizes reais e portanto f"'(x) tem pelo menos uma 
raiz real. 
 Mas f"'(x) = (2^x)*(ln2)^3 = 0 
implica 2^x = 0, o que eh absurdo.
 []s

- Original Message - 

  From: 
  Rafael 
  To: [EMAIL PROTECTED] 
  Sent: Sunday, June 27, 2004 12:53 
AM
  Subject: Re: [obm-l] OBM - 1997
  
  O jeito mais fácil de se resolver essa equação é 
  graficamente.
  
  As soluções são x = - 0,7666... ou x = 2 ou x = 
  4.
  
  
  
- Original Message - 
From: 
[EMAIL PROTECTED] 
To: [EMAIL PROTECTED] 
Sent: Sunday, June 27, 2004 12:11 
AM
Subject: [obm-l] OBM - 1997
Ola 
pessoal O numero de solucoes reais da equacao x^2 = 2^x eh: 
a)0 b)1 c)2 d)3 e)4 



Re: [obm-l] OBM - 1997

2004-06-27 Por tôpico Marcio Afonso A. Cohen



 É bem simples formalizar que 
essa equação só tem uma raiz negativa (2^x - x^2 eh crescente e vem de -oo até 
1), mas nao eh tao obvio assim a formalizacao de que no total só temos tres 
raizes. Uma maneira eh a q se segue:
 Seja f(x) = 2^x - x^2. 
Como f(-1) = 1/2 - 1  0 e f(0) = 1  0, f tem ao menos uma raiz em 
(-1,0). Alem disso, f(2)=f(4)=0, logo f tem pelo menos 3 raizes.
 Suponha por absurdo que f tenha 
pelo menos 4 raizes reais x1,x2,x3,x4. Pelo teorema de Rolle, f'(x) tem ao menos 
3 raizes reais (em (x1,x2),(x2,x3),(x3,x4)). Analogamente, o teorema de Rolle 
garante que f" tem ao menos 2 raizes reais e portanto f"'(x) tem pelo menos uma 
raiz real. 
 Mas f"'(x) = (2^x)*(ln2)^3 = 0 
implica 2^x = 0, o que eh absurdo.
 []s

- Original Message - 

  From: 
  Rafael 
  To: [EMAIL PROTECTED] 
  Sent: Sunday, June 27, 2004 12:53 
AM
  Subject: Re: [obm-l] OBM - 1997
  
  O jeito mais fácil de se resolver essa equação é 
  graficamente.
  
  As soluções são x = - 0,7666... ou x = 2 ou x = 
  4.
  
  
  
- Original Message - 
From: 
[EMAIL PROTECTED] 
To: [EMAIL PROTECTED] 
Sent: Sunday, June 27, 2004 12:11 
AM
Subject: [obm-l] OBM - 1997
Ola 
pessoal O numero de solucoes reais da equacao x^2 = 2^x eh: 
a)0 b)1 c)2 d)3 e)4 



Re: [obm-l] Desigualdade

2004-06-07 Por tôpico Marcio Afonso A. Cohen
Inicialmente, veja que a desigualdade das medias potenciais M(7)=M(5)
implica (x^7 + y^7) = 2[(x^5 + y^5)/2]^(7/5), de modo que:
(x^7+y^7)/(x^5+y^5) = [(x^5+y^5)/2]^(2/5) = [(x+y)/2]^2  (na ultima
passagem usei M(5)=M(1))
Portanto, LE = [(a+b)/2]^2 + [(c+b)/2]^2 + [(a+c)/2]^2
Mas, por Cauchy, (u^2 + v^2 + w^2)*(1+1+1) = u+v+w, donde:
LE = [(a+b)/2 + (c+b)/2 + (a+c)/2]/3 = 1/3  (apenas aqui usei a+b+c=1).
Abracos,
Marcio

- Original Message -
From: Maurizio [EMAIL PROTECTED]
To: [EMAIL PROTECTED]
Sent: Thursday, June 03, 2004 4:18 PM
Subject: [obm-l] Desigualdade


 Alguém saberia resolver esta desigualdade:

 Se a+b+c=1, prove que:

 (a^7+b^7)/(a^5+b^5)+(b^7+c^7)/(b^5+c^5)+(c^7+a^7)/(c^5+a^5) = 1/3
 =
 Instruções para entrar na lista, sair da lista e usar a lista em
 http://www.mat.puc-rio.br/~nicolau/olimp/obm-l.html
 =


=
Instruções para entrar na lista, sair da lista e usar a lista em
http://www.mat.puc-rio.br/~nicolau/olimp/obm-l.html
=


[obm-l] Convergencia

2004-05-30 Por tôpico Marcio Afonso A. Cohen



 Dado um vetor u =(u1, u2, ..., 
un) de coordenadas reais nao negativas, define-se T(u) da seguinte forma: 
Pegamos u_i,a menor das coordenadas de u e u_j, a maior delas (em ambos os 
casos, se houver mais de um, pega-se o de menor indice) e trocamos ambas pela 
média (u_i + u_j)/2. 
 Mostre que a sequencia definida 
por x_(n+1) = T(x_n), x_0=uconverge, ou entao exiba um 
contra-exemplo.
 []s
 Marcio
 


Re: [obm-l] Resultado da OIMU

2004-05-26 Por tôpico Marcio Afonso A. Cohen
Do ita... Em geral saia sempre a universidade de cada premiado..  Isso
responderia sua pergunta :)  A cidade ao lado representa onde a pessoa fez a
prova.. Essas pessoas sao, se nao me engano, passaram a maior parte de suas
vidas em goiás, rio, goiás, rio, fortaleza, rio, fortaleza, fortaleza,
paraná (???) e fortaleza respectivamente. As faculdades na qual estudavam na
epoca da prova eram ita, ime, ita, ufrj, ufc, ime, ime, ita (???), ita, ita.
[]s
Marcio

- Original Message -
From: Sergio Lima Netto [EMAIL PROTECTED]
To: [EMAIL PROTECTED]
Sent: Wednesday, May 26, 2004 11:09 AM
Subject: Re: [obm-l] Resultado da OIMU


 De onde vem toda esta forca
 de Sao Jose dos Campos?
 Que bacana. Parabens a todos
 (organizadores, participantes e premiados)
 Abraco,
 sergio



 On Wed, 26 May 2004, Olimpiada Brasileira de Matematica wrote:

  Caros(as) amigos(as) da lista:
 
  Finalmente publicamos o resultado da VI OIMU
  no site da OBM.
  Confiram também as provas, soluções e resultados
  internacionais.
 
  Abraços, Nelly.
 
 
  Resultado Brasileiro:
 
  Humberto Silva Naves Ouro (S.J. dos Campos - SP)
  Marcio Afonso Assad Cohen Prata (Rio de Janeiro - RJ)
  Carlos Stein Naves de Brito Prata (S.J. dos Campos - SP)
  Bernardo Freitas Paulo da Costa Bronze (Rio de Janeiro - RJ)
  Yuri Gomes Lima Bronze (Fortaleza - CE)
  Marcos Francisco Ferreira Martinelli Bronze (Rio de Janeiro - RJ)
  Daniele Veras de Andrade Bronze (Rio de Janeiro - RJ)
  Einstein dos Nascimento Júnior Menção (Fortaleza - CE)
  Estillac Lins Maciel Borges Filho Menção (S.J. dos Campos - SP)
  Thiago da Silva Sobral Menção (S.J. dos Campos - SP)
 
 

 =
 Instruções para entrar na lista, sair da lista e usar a lista em
 http://www.mat.puc-rio.br/~nicolau/olimp/obm-l.html
 =


=
Instruções para entrar na lista, sair da lista e usar a lista em
http://www.mat.puc-rio.br/~nicolau/olimp/obm-l.html
=


Re: [obm-l] Cone Sul - Problema 2

2004-05-25 Por tôpico Marcio Afonso A. Cohen
Nao cheguei a escrever no papel, mas essa solucao parece estar perfeita.
Legal! Foi a solucao do Leandro, de Fortaleza, uma das mais bonitas da
prova. Eu fiz uma solucao bem mais feinha soh pra mostrar pra eles que dava
pra fazer por complexos sem nem desenhar a figura..
Legal!! Tentem o 3 e o 6..
Abracos,
Marcio


- Original Message -
From: André Araújo [EMAIL PROTECTED]
To: [EMAIL PROTECTED]
Sent: Tuesday, May 25, 2004 1:17 PM
Subject: RE: [obm-l] Cone Sul - Problema 2


 Abaixo uma outra solucao p/ o problema 2 da Cone Sul.

 segunda solucao:

 Seja S a intersecao de AB com a reta PO, onde O eh o centro de C. Eh facil
 ver q AB eh perpendicular a PS. Dai conclui-se:

 i) quadrilatero PMSA eh inscritivel (ang PSA = ang PMA = 90);
 ii) quadrilatero PNSB eh inscritivel (ang PSB = ang PNB = 90);

 Como PA e PB sao tangentes a C, tem-se:

 iii) ang ABQ = ang PAM = arco menor AQ/2;
 iv) ang PBQ = ang BAQ = arco menor BQ/2;

 De i), ii), iii) e iv) tem-se:

 v) ang MSP = ang PAM = ang ABQ = ang NPS;
 vi) ang NSP = ang PBQ = ang BAQ = ang MPS;

 De v) e vi) conclui-se que os triangulos PMS e PNS sao congruentes, caso
 A.L.A. Ou seja, PMSN eh paralelogramo. Logo a reta MN corta o ponto medio
PS
 (fixo).

 [ ]'s

 AA.



 Cone Sul - Problema 2
 
 Dada uma circunferencia C e um ponto P exterior a ela, tracam-se por P
as
 duas tangentes aa circunferencia, sendo A e B os pontos de tangencia.
 Toma-se um ponto Q sobre o menor arco AB de C. Seja M a intersecao da
reta
 AQ com a perpendicular a AQ tracada por P e seja N a intersecao da reta
BQ
 com a perpendicular a BQ tracada por P. Demonstre que, ao variar Q no
arco
 AB, todas as retas MN passam por um mesmo ponto.
 
 
 Solucao:
 
 Sejam:
 H o pe da perpendicular de P a AB
 R e S as projecoes de N e M, respectivamente, a PH
 Q e T as projecoes de N e M, respectivamente, a AB
 
 No triangulo PNM:
 PN = PB.sen(PBN) (I)
 
 QH = NR = PN.sen(NPR) = PN.sen(NBA) (quadrilat. NPBH inscrit.) = (por
I)
 
 QH = PB.sen(PBN).sen(NBA)
 
 Da mesma forma encontramos:
 
 TH = PA.sen(PAM).sen(MAB)
 
 Como PA = PB, PAM = NBA e PBN = MAB, entao GH = TH
 
 Logo, a intersecao de MN com a altura PH se da no ponto medio de MN, que
 chamamos de L, e LH eh base media do trapezio QNMT com bases NQ e MT.
 Entao LH = (NQ + MT)/2
 
 Mas NQ = PH - PR = PH - PN.cos(NPR) = PH - PB.sen(PBN).cos(NBA)
 
 Da mesma forma:
 
 MT = PH - PA.sen(NBA).cos(PBN)
 
 e
 
 NQ + MT = 2PH - PA.(sen(PBN).cos(NBA) + sen(NBA).cos(PBN)) =
 
 = 2PH - PA.sen(PBN + NBA) = 2PH - PA.sen(PBA) = 2PH - PH = PH
 
 e LH = (NQ + MT)/2 = PH/2
 
 
 Ou seja, todas as retas MN passam pelo ponto medio da altura PH.
 
 []'s
 
 #
 # MSc. Edson Ricardo de A. Silva#
 # Computer Graphics Group (CRAB)#
 # Federal University of Ceara (UFC) #
 #
 
 =
 Instruções para entrar na lista, sair da lista e usar a lista em
 http://www.mat.puc-rio.br/~nicolau/olimp/obm-l.html
 =

 _
 MSN Messenger: converse com os seus amigos online.
 http://messenger.msn.com.br

 =
 Instruções para entrar na lista, sair da lista e usar a lista em
 http://www.mat.puc-rio.br/~nicolau/olimp/obm-l.html
 =


=
Instruções para entrar na lista, sair da lista e usar a lista em
http://www.mat.puc-rio.br/~nicolau/olimp/obm-l.html
=


[obm-l] Re: [obm-l] Questão de 2o. grau

2004-05-09 Por tôpico Marcio Afonso A. Cohen



 Em parte. Tudo que voce diz eh 
verdade, mas eu exigiria uma explicacao um pouquinho melhor de pq n! eh maior 
que n^2. Mas a ideia eh otima e funciona. Eu acho qfaria algo como: p/ 
n3,1! + 2! + ... + n! = n! + (n-1)!+1  n(n-1) + (n-1) + 1 = 
n^2.
 Uma outra opcao eh olhar mod 
10.

  - Original Message - 
  From: 
  Fellipe Rossi 
  To: [EMAIL PROTECTED] 
  Sent: Sunday, May 09, 2004 5:42 PM
  Subject: [obm-l] Questão de 2o. 
grau
  
  Como vocês demonstrariam, para 2o. grau, que 
  
  
  para n=1, n pertence a Z. apenas n=1 e n=3 
  são raízes da equação:
  
  1!+2!+3!+...+n! = n^2 
  
  Vocês aceitariam uma resolução que mostrasse, com 
  exemplos (4!=24, 4^2=16 ; 5!=120, 5^2=25, e assim por diante...) que para 
  n=4. n! é maior que n^2 e que como o lado esquerdo da igualdade eh 
  n!+valor positivo, ela vai ser sempre maior que o lado direito para n=4, e 
  substituindo n por 1, 2 e 3 chegamos q apenas 1 e 3 são raizes?
  
  Essa qustão caiu, se não me engano, na prova 
  específica da UFRJ 1992.
  
  Abraços! 
  Rossi


[obm-l] Re: [obm-l] Exercício

2004-04-24 Por tôpico Marcio Afonso A. Cohen



 Tome r = (r1+r2)/2. Acho que nao 
era exatamente esse o enunciado que voce queria :)

  - Original Message - 
  From: 
  Marcelo 
  Augusto Pereira 
  To: [EMAIL PROTECTED] 
  Sent: Saturday, April 24, 2004 6:20 
  PM
  Subject: [obm-l] Exercício
  
  Mostrar que se r1 e r2 são racionais e r1r2, 
  então existe um racional r tal que 
r1rr2.


Re: [obm-l] En: Putnam Question

2004-03-14 Por tôpico Marcio Afonso A. Cohen
Em geral esse tipo de questão pede por uma resposta que envolva apenas
os dados do enunciado.. Quando se deseja que a resposta fique em função de
BC por exemplo, é comum dizer algo como Conidere BC = a..
Num outro email foi discutida a opcao de se usar trigonometria em
problemas de geometria. A sugestao de se usar trigonometria, bem como
vetores, numeros complexos e qualquer outra arma eh, no meu ver, excelente.
Esse eh um otimo caso para se usar vetores.. Eu escrevi essa solucao com
o auxilio de papel e caneta para calcular os 3 determinantes, e portanto
sugiro que qualquer um que tente acompanha-la nos detalhes faca o mesmo:

Inicialmente note que nao ha perda de generalidade em supor A = (0,0), B
= (1,0), C=(0,1) e depois dividir a resposta por 1/2 (*).
Seja pois F = (0,t) um pto do lado AC, e G=(k,0) um do lado AB.
Do enunciado, R eh medio de BF = R = (1/2, t/2) e portanto como E eh a
intersecao da reta AR:y=tx com BC: x+y=1 temos E = (1/(t+1) , t/(t+1) ).
S eh medio de CG, logo S = (k/2, 1/2) e como B,S,F estao alinhados, o
determinante nos da 2t=kt+1 (i).
O ponto medio de AE eh T = ( 1/[2(t+1)], t/[2(t+1)] ), e como C,T,G
estao alinhados, o det nos da: 2k=1-kt (ii).
De (i) + (ii), t+k = 1 e substituindo em (i), t^2+t-1=0 (iii), donde
como 0t1, t = [sqrt(5)-1]/2 (iv).
O dobro da area do triangulo RST eh dada (calculando o det com R,S,T)
por:
t*(1-kt) / [4*(t+1)] que eh igual a t*(1-t)/[2*(t+1)] por (i).
Multiplicando em cima e em baixo por 1-t e usando (iii), vem Resp =
[t(1-t)^2] / [2*(1-t^2)] = (1-t)^2 / 2 = t^4/2.
Usando (iv), concluimos portanto que a area pedida vale [7-3sqrt(5)]/4.

(*) Dado um triangulo qualquer, tome um transformacao afim (T(v)= A*v+B, com
det(A)!=0) que leve ele no meu triangulo ABC. Não é difícil ver que esse
tipo de transformacao preserva incidência, razão entre segmentos paralelos e
razão entre áreas, de forma que eh suficiente resolver o caso acima
exposto).

Marcio.

- Original Message -
From: Rafael [EMAIL PROTECTED]
To: [EMAIL PROTECTED]
Sent: Sunday, March 14, 2004 4:11 PM
Subject: Re: [obm-l] En: Putnam Question


 Pelo visto, hoje alguém estava (des)inspirado para comentar as
mensagens...

 Perdoe-me, em que trecho do enunciado diz-se que há valor numérico para a
 resposta? Vou ajudá-lo, eis o enunciado:

 Triangle ABC has an area 1. Points E, F, and G,lie respectively on sides
BC
 , CA, and AB, such that AE bisects BF at point R, BF bisects CG at point
S,
 and CG bisects AE at point T, Find the area of triangle RST.

 Concordo que a área de ABC é dada, mas a questão não *pede* um valor
 numérico para o triângulo RST, isso não está escrito em qualquer lugar. A
 menos que, além de dizer que o exercício está resolvido errado, você possa
 resolver corretamente, com ou sem valores numéricos

=
Instruções para entrar na lista, sair da lista e usar a lista em
http://www.mat.puc-rio.br/~nicolau/olimp/obm-l.html
=


[obm-l] Re: [obm-l] Re: [obm-l] pertinência

2004-03-10 Por tôpico Marcio Afonso A. Cohen
Continuo achando que a letra (a) eh verdadeira (embora ache que esse
tipo de questao nao seja lah tao importante). Por exemplo, dado S'={{},
1,3,{5},{7,8}}, seria bem natural dizer que {} pertence a S.  Retirando o {}
de S', obteriamos o conjunto S dado e faria sentido dizer que {} nao
pertence a S.
O vazio eh um conjunto, mas nada impede que usemos o simbolo de pertence
com ele... Assim como usamos para dizer que {5} pertence a S.. O {5} é um
conjunto, que nesse caso está sendo encarado como um elemento de S. Aliás,
essa é uma das razões que vejo para que utilizemos dois símbolos (contido e
pertence): evitar ambiguidades... {} pertence a S e {} contido em S sao
ambas afirmacoes validas e com significados diferentes.
A letra (d) tambem esta certa, como voce corretamente explicou num outro
email.
[]'s Marcio.



- Original Message -
From: Rafael [EMAIL PROTECTED]
To: [EMAIL PROTECTED]
Sent: Wednesday, March 10, 2004 1:58 AM
Subject: [obm-l] Re: [obm-l] pertinência


 Korshinói,

 A alternativa (a) indica que o conjunto vazio, representado por {}, não
 *pertence* ao conjunto S. Isso é falso por dois motivos. Primeiramente,
pois
 a relação entre conjuntos é de continência, e não pertinência. E, em
segundo
 lugar, o conjunto vazio está contido em qualquer conjunto. A demonstração
é
 simples: se assim não fosse, existiria pelo menos um elemento x que
 pertencesse ao {} de modo que x não pertenceria a S e isto nunca ocorre,
 pois não existe x de modo que x pertença a {}, logo, {} está contido em S.


 Abraços,

 Rafael de A. Sampaio




 - Original Message -
 From: [EMAIL PROTECTED]
 To: [EMAIL PROTECTED]
 Sent: Wednesday, March 10, 2004 1:10 AM
 Subject: [obm-l] pertinência


 Observem esse teste, onde só uma resposta tem que ser correta:
 Seja S={1,3,{5},{7,8}}. É correto afirmar que:
 a) { } não pertence a S.
 b) 3 não pertence a S.
 c) {7,8} está contido em S.
 d) {3,{5}} está contido em S.
 e) {5} não pertence a S.

 Obviamente, a respsota d está correta... mas... por que a resposta a não
 estaria, já que um conjunto pode ser elemento de um conjunto?

 Obrigado,
   Korshinói

 =
 Instruções para entrar na lista, sair da lista e usar a lista em
 http://www.mat.puc-rio.br/~nicolau/olimp/obm-l.html
 =


=
Instruções para entrar na lista, sair da lista e usar a lista em
http://www.mat.puc-rio.br/~nicolau/olimp/obm-l.html
=


[obm-l] Re: [obm-l] Re: [obm-l] Re: [obm-l] Re: [obm-l] pertinência

2004-03-10 Por tôpico Marcio Afonso A. Cohen
Exatamente, isso resume um pouco o que eu escrevi... Por isso a opção (a)
também é correta. Ela afirma justamente que o vazio não é um elemento de S.

- Original Message -
From: Paulo Rodrigues [EMAIL PROTECTED]
To: [EMAIL PROTECTED]
Sent: Wednesday, March 10, 2004 10:04 AM
Subject: [obm-l] Re: [obm-l] Re: [obm-l] Re: [obm-l] pertinência


 Se {} fosse um elemento de S, este conjunto teria 5 elementos e não 4.


 -Mensagem Original-
 De: Marcio Afonso A. Cohen [EMAIL PROTECTED]
 Para: [EMAIL PROTECTED]
 Enviada em: quarta-feira, 10 de março de 2004 08:36
 Assunto: [obm-l] Re: [obm-l] Re: [obm-l] pertinência


 Continuo achando que a letra (a) eh verdadeira (embora ache que esse
 tipo de questao nao seja lah tao importante). Por exemplo, dado S'={{},
 1,3,{5},{7,8}}, seria bem natural dizer que {} pertence a S.  Retirando o
{}
 de S', obteriamos o conjunto S dado e faria sentido dizer que {} nao
 pertence a S.
 O vazio eh um conjunto, mas nada impede que usemos o simbolo de
pertence
 com ele... Assim como usamos para dizer que {5} pertence a S.. O {5} é um
 conjunto, que nesse caso está sendo encarado como um elemento de S. Aliás,
 essa é uma das razões que vejo para que utilizemos dois símbolos (contido
e
 pertence): evitar ambiguidades... {} pertence a S e {} contido em S
sao
 ambas afirmacoes validas e com significados diferentes.
 A letra (d) tambem esta certa, como voce corretamente explicou num
outro
 email.
 []'s Marcio.



 - Original Message -
 From: Rafael [EMAIL PROTECTED]
 To: [EMAIL PROTECTED]
 Sent: Wednesday, March 10, 2004 1:58 AM
 Subject: [obm-l] Re: [obm-l] pertinência


  Korshinói,
 
  A alternativa (a) indica que o conjunto vazio, representado por {}, não
  *pertence* ao conjunto S. Isso é falso por dois motivos. Primeiramente,
 pois
  a relação entre conjuntos é de continência, e não pertinência. E, em
 segundo
  lugar, o conjunto vazio está contido em qualquer conjunto. A
demonstração
 é
  simples: se assim não fosse, existiria pelo menos um elemento x que
  pertencesse ao {} de modo que x não pertenceria a S e isto nunca ocorre,
  pois não existe x de modo que x pertença a {}, logo, {} está contido em
S.
 
 
  Abraços,
 
  Rafael de A. Sampaio
 
 
 
 
  - Original Message -
  From: [EMAIL PROTECTED]
  To: [EMAIL PROTECTED]
  Sent: Wednesday, March 10, 2004 1:10 AM
  Subject: [obm-l] pertinência
 
 
  Observem esse teste, onde só uma resposta tem que ser correta:
  Seja S={1,3,{5},{7,8}}. É correto afirmar que:
  a) { } não pertence a S.
  b) 3 não pertence a S.
  c) {7,8} está contido em S.
  d) {3,{5}} está contido em S.
  e) {5} não pertence a S.
 
  Obviamente, a respsota d está correta... mas... por que a resposta a não
  estaria, já que um conjunto pode ser elemento de um conjunto?
 
  Obrigado,
Korshinói
 
 
=
  Instruções para entrar na lista, sair da lista e usar a lista em
  http://www.mat.puc-rio.br/~nicolau/olimp/obm-l.html
 
=
 

 =
 Instruções para entrar na lista, sair da lista e usar a lista em
 http://www.mat.puc-rio.br/~nicolau/olimp/obm-l.html
 =

 =
 Instruções para entrar na lista, sair da lista e usar a lista em
 http://www.mat.puc-rio.br/~nicolau/olimp/obm-l.html
 =


=
Instruções para entrar na lista, sair da lista e usar a lista em
http://www.mat.puc-rio.br/~nicolau/olimp/obm-l.html
=


[obm-l] Re: [obm-l] Lema para funções contínuas

2004-03-08 Por tôpico Marcio Afonso A. Cohen
Sim. Como voce fala em derivada para todo x, a hipotese de continuidade
na verdade deve ser de diferenciabilidade. Seja h(x) = f(x) - g(x). Entao,
h'(x)  0 sempre, donde h eh uma funcao estritamente crescente, de modo que
a equacao h(x) = 0 pode ter no maximo uma solucao. Nao vejo a necessidade de
se ter f crescente para esse resultado. Agora, se o dominio e o
contra-dominio realmente sao esses que voce mencionou, entao o unico valor
possivel para h^-1(0) eh o proprio 0...
Marcio


- Original Message -
From: Rafael [EMAIL PROTECTED]
To: OBM-L [EMAIL PROTECTED]
Sent: Monday, March 08, 2004 3:21 AM
Subject: [obm-l] Lema para funções contínuas


 Pessoal,

 Estava visitando alguns sites na internet e li isso:

 Se f e g são funções contínuas (f,g : R+ -- R+), com f crescente, tais
 que: f'(x)  g'(x) para todo x real positivo, então o número de soluções
da
 equação f(x) = g(x) é no máximo um.

 Isso é verdade?


 Obrigado,

 Rafael de A. Sampaio

 =
 Instruções para entrar na lista, sair da lista e usar a lista em
 http://www.mat.puc-rio.br/~nicolau/olimp/obm-l.html
 =


=
Instruções para entrar na lista, sair da lista e usar a lista em
http://www.mat.puc-rio.br/~nicolau/olimp/obm-l.html
=


[obm-l] arccos((raiz(5)-1)/2)

2004-02-29 Por tôpico Marcio Afonso A. Cohen



 Bom gente, eu mandei esse 
problema pra lista, acompanhei os emails do Arthur, do Cláudio e do Nicolau 
sobre ele, inclusive chegando a solucao final. Legal. Segue abaixo uma outra 
solucao, bastante interessante, para o problema. (o fanático por 
polinomios de chebyshev da lista vai adorar :) ).

 A ideia eh que se x = arccos((raiz(5)-1)/2) fosse 
multiplo racionalde Pi, entao haveria um inteiro n tal que cos(nx) = 0, e 
portanto cos(x)seria raiz de uma equacao de coeficientes inteiros t^n + 
...t^n-1 +... = 0(polinomio de chebyshev). Mas por um lado todas as raizes 
desse polinomioestao em [-1,1] (afinal, temos cos(nx) = t^n + ... e 
isso vale zero paranx(k) = pi/2 + kpi, k = 0, 1, 2, 3, ..., n-1, o que ja 
lista todas as raizesdo polinomio como sendo cos x(k) para algum k), e por 
outro lado oconjugadode cosx, (raiz(5)+1)/2  1 tambem deveria ser 
raiz dela... Legal né?
 Abracos,
 Marcio 
Cohen.


Re: [obm-l] arccos((raiz(5)-1)/2)

2004-02-29 Por tôpico Marcio Afonso A. Cohen



Onde está inteiro, leia-se racionais.

  - Original Message - 
  From: 
  Marcio Afonso A. Cohen 
  To: [EMAIL PROTECTED] 
  Sent: Thursday, January 29, 2004 11:09 
  PM
  Subject: [obm-l] 
  arccos((raiz(5)-1)/2)
  
   Bom gente, eu mandei esse 
  problema pra lista, acompanhei os emails do Arthur, do Cláudio e do Nicolau 
  sobre ele, inclusive chegando a solucao final. Legal. Segue abaixo uma outra 
  solucao, bastante interessante, para o problema. (o fanático por 
  polinomios de chebyshev da lista vai adorar :) ).
  
   A ideia eh que se x = arccos((raiz(5)-1)/2) fosse 
  multiplo racionalde Pi, entao haveria um inteiro n tal que cos(nx) = 0, 
  e portanto cos(x)seria raiz de uma equacao de coeficientes INTEIROS t^n 
  + ...t^n-1 +... = 0(polinomio de chebyshev). Mas por um lado todas as 
  raizes desse polinomioestao em [-1,1] (afinal, temos cos(nx) = t^n + 
  ... e isso vale zero paranx(k) = pi/2 + kpi, k = 0, 1, 2, 3, ..., 
  n-1, o que ja lista todas as raizesdo polinomio como sendo cos x(k) para 
  algum k), e por outro lado oconjugadode cosx, (raiz(5)+1)/2  1 
  tambem deveria ser raiz dela... Legal né?
   Abracos,
   Marcio 
Cohen.


Re: [obm-l] Unificando Geometrias

2004-02-29 Por tôpico Marcio Afonso A. Cohen
Oi Álvaro! Eu dei uma aula justamente sobre a aplicação de numeros
complexos na geometria euclidiana na ultima semana olimpica. Voce pode
baixar o conteudo da aula em www.obm.org.br . Qualquer dificuldade em
resolver os problemas, pode perguntar. Vale a pena tambem ver o artigo do
Edmilson, publicado na Eureka 6 sobre esse tema.
Um livro bastante bom sobre isso (na verdade, o unico que eu conheco)
chama-se Complex Numbers and Geometry, de Liang-shin Hahn.
Sobre aplicacoes de Geometria Analitica tradicional na geometria, eu
infelizmente ainda nao conheco nenhum livro especifico.. O livro Problems
solving through problems, de Loren Larson, traz uma seção dedicada a isso,
mas eh pouca coisa..
Abracos,
Marcio Cohen.

- Original Message -
From: Alvaro de Jesus Netto [EMAIL PROTECTED]
To: [EMAIL PROTECTED]
Sent: Monday, March 01, 2004 1:29 AM
Subject: [obm-l] Unificando Geometrias


 Saudações.
 Procuro sugestões sobre um bom livro que utilize Geometria Analítica e/ou
 Números Complexos para resolver problemas de Geometria Euclidiana.
 Gostaria também de saber se existe algum software (de preferência para
Linux)
 que desenhe figuras geométricas utilizando-se de Geometria Analítica ou
não e
 que salve em formatos usuais como jpg ou gif.
 Agradeço a atenção.
 --
 Alvaro de Jesus Netto.
 =
 Instruções para entrar na lista, sair da lista e usar a lista em
 http://www.mat.puc-rio.br/~nicolau/olimp/obm-l.html
 =


=
Instruções para entrar na lista, sair da lista e usar a lista em
http://www.mat.puc-rio.br/~nicolau/olimp/obm-l.html
=


[obm-l] Re: [obm-l] Soluções da obm-u

2004-02-19 Por tôpico Marcio Afonso A. Cohen
Oi Domingos, no meu último email para essa lista eu mostrei que se a e b
sao algebricos, entao a+b e ab tmb sao, adaptando a ideia que o Carlos usou
para resolver a questao 5 da obm-u do ano passado.. De uma lida nesse email
e tente adaptar (note que eh muito parecido dizer que a satisfaz a^n +
p1*a^n-1 + ... pn = 0 com (pi)'s racionais e dizer que uma funcao f satisfaz
f^n + p1*f^n-1 + ... + pn com (pi)'s funcoes racionais e f^k sendo a k-esima
derivada), pq a coisa eh essencialmente a mesma. Se vc nao conseguir,
pergunte que eu completo os detalhes!
Abraços,
Marcio.


- Original Message -
From: Domingos Jr. [EMAIL PROTECTED]
To: [EMAIL PROTECTED]
Sent: Wednesday, February 18, 2004 3:16 PM
Subject: [obm-l] Soluções da obm-u


 Alguém aí tem as soluções da obm-u do ano passado?
 Estou procurando mais especificamente pelo problema 5.

 [ ]'s

 =
 Instruções para entrar na lista, sair da lista e usar a lista em
 http://www.mat.puc-rio.br/~nicolau/olimp/obm-l.html
 =


=
Instruções para entrar na lista, sair da lista e usar a lista em
http://www.mat.puc-rio.br/~nicolau/olimp/obm-l.html
=


Re: [obm-l] Numeros algebricos e transcendentes

2004-02-12 Por tôpico Marcio Afonso A. Cohen
Oi Arthur, tudo bem?
Eu vi a demonstracao de que os nrs algebricos sao fechados para soma e
multiplicacao num livro chamado Numeros irracionais e transcedentes, de
Djairo Figueiredo. Eu achei o livro interessante, pois nele eu vi pela
primeira vez a demonstracao de que Pi era transcendente (alem de
demonstracoes para transcedencia de 'e' e irracionalidade de 'Pi' e 'e').
Isso resolve o seu problema, pq se x eh algebrico e y eh transcendente,
entao x+y nao pode ser algebrico (pq se fosse teriamos y = (x+y) + (-x)
algebrico pelo resultado acima). O caso da multiplicacao eh analogo.
Eu fiquei muito chateado de nao ter pensado nisso durante a prova da
OBM-u de 2003, pois a questao 5 da prova usava uma ideia semelhante à ideia
dessas demonstracoes. Inclusive, o que eu estou escrevendo aqui nao foi
retirado diretamente do livro, mas sim do que eu lembro das ideias
principais que o Carlos (Stein) usou para resolver a questao na prova..
Nao sei de um site onde tenha essa demonstracao, mas a ideia eh a
seguinte:
Suponha que x e y sao algebricos. Entao, existem n-uplas
(x_1,x_2,...,x_n) e (y_1, ..., y_n) tais que x_1 + x*x_2 + ... + (x^n-1)*x_n
= 0. Em particular, podemos escrever x^n em funcao de 1,x,x^2,...,x^n-1,
sendo cada coeficiente uma combinacao linear dos racionais (x_1, ..., x_n).
Tambem podemos fazer isso para x^n+1, x^n+2, e assim por diante, e podemos
fazer algo analogo para y.
 Olhe agora para os numeros 1, (x+y), (x+y)^2, (x+y)^3, ... .
Pelo que foi visto acima, veja que cada termo x^p * y^q pode ser escrito
como uma combinacao linear dos termos x^i * y^j para i,j em {0,1,...,n-1}
(basta escrevermos x^p em funcao de 1,x,x^2,...,x^n-1 e fazer analogo para
y^q. Portanto cada um dos termos (x+y)^k pode ser escrito como uma
combinacao racional dos mn numeros x^i * y^j acima descritos. Ou seja, a
cada (x+y)^k associamos um vetor de mn componentes racionais (r_ k1, r_k2,
...).
Pegando k=1,2,...,mn+1, obtemos mn+1 vetores do espaco Q^mn (com
escalares em Q), e portanto eles devem ser linearmente dependentes, ou seja,
existem racionais q_1, ... nao todos nulos de modo que q_1*(x+y) +
q_2*(x+y)^2 + ... + q_n*(x+y)^n = 0 (pq cada componente do vetor será zero,
e portanto essa soma sera zero).
Abracos,
Marcio


- Original Message -
From: Artur Costa Steiner [EMAIL PROTECTED]
To: [EMAIL PROTECTED]
Sent: Thursday, February 12, 2004 10:20 AM
Subject: [obm-l] Numeros algebricos e transcendentes


 Alguem poderia indicar algum material ou algum site sobre numeros
algebricos
 e transcendentes?
 Especificamente, alguem tem uma demonstracao de que a soma de um
 transcendente com um  algebrico eh trancendente e o produto de um
 transcendente por um algebrico nao nulo eh transcendente?
 Obrigado
 Artur

 
 OPEN Internet
 @ Primeiro provedor do DF com anti-vírus no servidor de e-mails @


 =
 Instruções para entrar na lista, sair da lista e usar a lista em
 http://www.mat.puc-rio.br/~nicolau/olimp/obm-l.html
 =


=
Instruções para entrar na lista, sair da lista e usar a lista em
http://www.mat.puc-rio.br/~nicolau/olimp/obm-l.html
=


[obm-l] Problema legal

2004-02-08 Por tôpico Marcio Afonso A. Cohen



 Mostre 
que o numero [arccos((sqrt(5)-1)/2 )] / pi eh 
irracional.


Re: [obm-l] Duvida - Matriz inversivel

2004-02-08 Por tôpico Marcio Afonso A. Cohen



 Bom, o Morgado e o Claudio ja 
comentaram que o que se pode afiramar eh que BA eh sempre nao inversivel. Isso 
ja foi inclusive provado aqui na lista se nao em engano. Esse problema ja caiu, 
dessa forma, no vestibular do IME, e de uma forma mais generica no vestibular da 
Unicamp. Uma solucao legal, pouco bracal e sem usar muita algebra linear, eh 
voce completar as matrizes A e B com zeros de modo que fiquem ambas 
4x4:
 A' =[a b c d, e f g 
h, 0 0 0 0, 0 0 0 0], B' = [i j 0 0, k l 0 0, m n 0 0, o p 0 0]
 E entao notarque BA = 
B'A', e det(B'A') = det(B')*det(A') = 0*0 = 0, donde BA nao eh inversivel 
nunca.

 

  - Original Message - 
  From: 
  João 
  Silva 
  To: [EMAIL PROTECTED] 
  Sent: Sunday, February 08, 2004 11:04 
  AM
  Subject: [obm-l] Duvida - Matriz 
  inversivel
  
  Alguem sabe como se resolve:
  
  - Seja A uma matriz 2 X 4 e B uma matriz 4 X 2, ambas matrizes de 
  elementos inteiros. Verifique se a matriz AB é inversível.
  
  
  Yahoo! 
  GeoCities: 15MB de espaço grátis para criar seu web 
site!


Re: [obm-l] Duvida - Matriz inversivel

2004-02-08 Por tôpico Marcio Afonso A. Cohen



 Bom, o Morgado e o Claudio ja 
comentaram que o que se pode afiramar eh que BA eh sempre nao inversivel. Isso 
ja foi inclusive provado aqui na lista se nao em engano. Esse problema ja caiu, 
dessa forma, no vestibular do IME, e de uma forma mais generica no vestibular da 
Unicamp. Uma solucao legal, pouco bracal e sem usar muita algebra linear, eh 
voce completar as matrizes A e B com zeros de modo que fiquem ambas 
4x4:
 A' =[a b c d, e f g 
h, 0 0 0 0, 0 0 0 0], B' = [i j 0 0, k l 0 0, m n 0 0, o p 0 0]
 E entao notarque BA = 
B'A', e det(B'A') = det(B')*det(A') = 0*0 = 0, donde BA nao eh inversivel 
nunca.

 

  - Original Message - 
  From: 
  João 
  Silva 
  To: [EMAIL PROTECTED] 
  Sent: Sunday, February 08, 2004 11:04 
  AM
  Subject: [obm-l] Duvida - Matriz 
  inversivel
  
  Alguem sabe como se resolve:
  
  - Seja A uma matriz 2 X 4 e B uma matriz 4 X 2, ambas matrizes de 
  elementos inteiros. Verifique se a matriz AB é inversível.
  
  
  Yahoo! 
  GeoCities: 15MB de espaço grátis para criar seu web 
site!


[obm-l] Problema legal

2004-02-08 Por tôpico Marcio Afonso A. Cohen



 Mostre 
que o numero [arccos((sqrt(5)-1)/2 )] / pi eh 
irracional.


Re: [obm-l] Tenista da obm

2004-01-06 Por tôpico Marcio Afonso A. Cohen
Na hora da prova eu interpretei errado o problema e acabei fazendo um
mais legal!! Pensem na seguinte variação desse problema:
Um tenista tem 30 dias para prepar-se para um torneio. Se ele treina 3 dias
seguidos, ele tem fadiga muscular e nao pode treinar o proximo. Ele entao
decide que irá treinar 20 dias, sem nunca treinar 4 seguidos. De quantas
maneiras diferentes ele pode escolher os 10 dias de descanso?

Minha solucao na prova nao foi muito linda, mas eh um problema legal de
se pensar (obvio que eu fui penalizado na prova por ter feito outro
problema... mas a ideia se aplica a esse tmb!)

Abracos,
Marcio

- Original Message -
From: Domingos Jr. [EMAIL PROTECTED]
To: [EMAIL PROTECTED]
Sent: Tuesday, January 06, 2004 1:38 PM
Subject: [obm-l] Re: [obm-l] CAMPEÕES!


 Um tenista tem 30 dias para preparar-se para um torneio. Se ele treina 3
 dias
 seguidos ele tem fadiga muscular. Ele, então, decide que, durante esses 30
 dias, irá treinar 20 dias, sem nunca treinar 3 dias seguidos, e descansar
 nos
 outros 10 dias. De quantas maneiras diferentes ele pode escolher os 10
dias
 de
 descanso?(OBM - Nível Universitário)


 Quebre os 30 dias em blocos de 3 dias.
 Quais são os possíveis padrões para esses blocos dado as restrições acima?
 Depois de formular os padrões, qual padrão pode suceder outro padrão?
 Respondendo essas perguntas você deve chegar na resposta.

 =
 Instruções para entrar na lista, sair da lista e usar a lista em
 http://www.mat.puc-rio.br/~nicolau/olimp/obm-l.html
 =


=
Instruções para entrar na lista, sair da lista e usar a lista em
http://www.mat.puc-rio.br/~nicolau/olimp/obm-l.html
=


[obm-l] Treinamento

2003-10-27 Por tôpico Marcio Afonso A. Cohen



 Nessa semana, excepcionalmente, 
nao havera reuniao de treinamento na terca feira no impa.
Abracos,
Marcio





[obm-l] 3 2's.

2003-10-23 Por tôpico Marcio Afonso A. Cohen



 Mostre como escrever qualquer 
inteiro n utilizando-seexatamente 3 algarismos, todos iguais a 2, e 
operações elementares (soma, subtração, multiplicação, divisão, log, 
exponencial, etc...).

 Por exemplo,1 = 2^(2-2), 2 
= 2+ 2 - 2, 3 = 2+(2/2), ..., generalize.
 
 Abraços, 

 Marcio
 


Re: [obm-l] Sistemas lineares

2003-10-21 Por tôpico Marcio Afonso A. Cohen



 Somando (2) e (3), x = (2+m)/2. 
Subtraindo-as, y = (2-m)/2. O sistema eh possivel sse essas equacoes satisfazem 
(1). Substituindo:
m(2+m) + (2-m) = 2 sse m^2 + m = 0 sse m=0 ou m=-1. 

 Para m diferente disso, o 
sistema é impossível (pois não há solução).
 []'s

  - Original Message - 
  From: 
  Nelson 
  To: [EMAIL PROTECTED] 
  Sent: Tuesday, October 21, 2003 7:14 
  PM
  Subject: [obm-l] Sistemas lineares
  
  Olá pessoal, gostaria de uma ajuda nessa questão.
  Discuta o sistema:
  (1)mx + y = 1
  (2)x + y = 2
  (3)x - y = m
  []´s Nelson
  
  
  Yahoo! 
  Mail - o melhor webmail do Brasil. Saiba 
  mais!


Re: [obm-l] 1o. dia- nivel U

2003-10-19 Por tôpico Marcio Afonso A. Cohen
Na verdade, nao acho que isso seja exatamente um problema da questao...
Se P eh o vertice da parabola, entao as retas do problema ficam
paralelas.. Pode-se dizer que elas se encontram num ponto da reta do
infinito por onde a nossa hiperbole tmb passa (veja a direcao da assintota).
Alem disso, se x_a = 0, a gente tem a equacao (num eixo especifico) xy' = 0,
que eh a hiperbole equilatera (rodando de pi/4) degenerada x^2 - y^2 = 0...


- Original Message -
From: Eduardo Casagrande Stabel [EMAIL PROTECTED]
To: [EMAIL PROTECTED]
Sent: Sunday, October 19, 2003 2:17 AM
Subject: [obm-l] 1o. dia- nivel U


 Olá Pessoal!

 Eu encontrei problemas da primeira questão. Enxerguei mal?

 1. Considere uma parábola e um ponto A fora dela, no plano. Para cada
ponto
 P da parábola sejam t a reta tangente em P, r a reta paralela ao eixo da
 parábola por P e Q o ponto de interseção de r com a perpendicular à t por
A.
 Provar que o ponto Q descreve uma hipérbole eqüilátera, com P percorrendo
a
 parábola.

 Há dois problemas. Se o ponto P é o vértice da parábola, não faz sentido
 falar em Q, pois ou as retas não se intersectam ou coincidem totalmente.
 Segundo, se o ponto A está sobre o eixo da parábola, então a figura
descrita
 não é uma hipérbola e sim uma reta! Acontece que a equação fica algo do
tipo
 y = y_A - 1/2a + x_a / x, colocando eixos coordenados (o Y no eixo da
 parábola e o X paralelo e passando pelo vértice da parábola). Sempre dá
uma
 hipérbole, a não ser no caso x_a = 0, quando dá uma espécie de hiperbole
 degenerada.

 Abraço, Duda.

 =
 Instruções para entrar na lista, sair da lista e usar a lista em
 http://www.mat.puc-rio.br/~nicolau/olimp/obm-l.html
 =


=
Instruções para entrar na lista, sair da lista e usar a lista em
http://www.mat.puc-rio.br/~nicolau/olimp/obm-l.html
=


Re: [obm-l] teo. simpsom

2003-10-12 Por tôpico Marcio Afonso A. Cohen



 Bom, a mensagem original nao eh 
minha, mas a ideia eh muito parecida com a sua solucao original. Seja P o ponto 
de encontro de duas das 4 circunferencias circunscritas. Pelo 
teorema da reta de simpson, as 4 projecoes de P nas retas dadas sao colineares 
(vc deve considera-las em dois grupos de 3 e usar que P esta em duas 
circunferencias). Agora, pela volta do teorema de simpson, voce conclui que P 
tambem esta nas outras duas circunferencias. (um desenho, ou pelo menos algumas 
letras, ajudaria bastante aqui).
 Abracos,
 Marcio

  - Original Message - 
  From: 
  A. C. 
  Morgado 
  To: [EMAIL PROTECTED] 
  Sent: Sunday, October 12, 2003 1:44 
  AM
  Subject: Re: [obm-l] teo. simpsom
  Estou interessado em conhecer essa sua soluçao.mparaujo 
  wrote:
  Se não me engano, a demonstração consta no Livro Geometria II do Prof. Wagner, juntamente com o Prof. Morgado e o Prof. Miguel Jorge. Este teorema garante que os pés das perpendiculares traçadas por um ponto P até as retas suportes dos lados de um triângulo estão alinhados se e só se o ponto P pertence a circunferência circunscrita ao triângulo e nesse caso a reta é chamada reta de Simson do triângulo relativamente ao ponto P.

Eu usei esse teorema pra resolver uma questão do IME que apareceu nessa lista e o Prof. Morgado resolveu usando o teorema de Miquel.

[]'s MP



=
  
De:"Isaac FJV" [EMAIL PROTECTED]
Para:"mat" [EMAIL PROTECTED]
Assunto:[obm-l] teo. simpsom








POR ACASO ALGUÉM CONHECE O TEOREMA DE 
SIMPSOM???






=
Instruções para entrar na lista, sair da lista e usar a lista em
http://www.mat.puc-rio.br/~nicolau/olimp/obm-l.html
=


  


Re: [obm-l] Problema da (segunda fase) obm-u 2002

2003-10-09 Por tôpico Marcio Afonso A. Cohen
Oi Domingos. Nao cheguei a ler a sua solucao toda, apenas dei uma olhada
em diagonal, mas ela parece estar certa. Inclusive, essa generalizacao foi
exatamente a solucao do Carlos na prova do ano passado (pelo que eu
conversei com ele), com uma abordagem extremamente parecida com a sua. Bem
legal.
Quanto a concentrar os esforcoes numa unica questao, minha estrategia em
geral eh dar uma esbocada em 2 ou ateh nos 3 problemas no inicio da prova
(umas meia hora) e depois concentrar meus esforcos no que eu acho que tenho
mais chances...
Mas as vezes eu julgo mal.. Por exemplo, na prova do ano retrasado
fiquei umas 3hs na questao de combinatoria (4) e ainda errei algumas
contas.. Depois, na 1h30m restante eu consegui fazer a questao 5..
Claramente dei sorte.. Se eu tivesse perdido mais tempo na 4, provavelmente
nao teria esquentado nem um pouco e teria deixado a 5 em branco, pq na
primeira lida eu julguei que ela era mais complicada (a minha sorte eh que
eu sabia que tinha que escrever algo nela, pq ela tinha uma letra a facil
que certamente valeria pontos).
Abracos,
Marcio

- Original Message -
From: Domingos Jr. [EMAIL PROTECTED]
To: [EMAIL PROTECTED]
Sent: Wednesday, October 08, 2003 2:58 PM
Subject: [obm-l] Problema da (segunda fase) obm-u 2002


 Olá!

 Estava vendo os problemas do ano passado (segunda fase) pra treinar pra
 segunda fase deste ano (eu passei!!!) e peguei pra resolver o problema 2,
de
 matrizes.
 Acho que consegui resolver uma generalização do problema... gostaria que o
 povo da lista desse uma olhada:

 http://www.linux.ime.usp.br/~domingos/obm-u-p2.pdf

 A propósito, qual seria a estratégia para a segunda fase, concentrar
 esforços num único problema? (se resolver um inteiro já é algo notável?)

 [ ]'s

 =
 Instruções para entrar na lista, sair da lista e usar a lista em
 http://www.mat.puc-rio.br/~nicolau/olimp/obm-l.html
 =


=
Instruções para entrar na lista, sair da lista e usar a lista em
http://www.mat.puc-rio.br/~nicolau/olimp/obm-l.html
=


[obm-l] Notas de Corte

2003-10-07 Por tôpico Marcio Afonso A. Cohen



 Acabaram de sair as notas de 
corte para a ultima fase no site da obm!
Marcio


Re: [obm-l] Sequencias de Cauchy

2003-10-01 Por tôpico Marcio Afonso A. Cohen
Porque o numero de termos eh arbitrariamente grande.

- Original Message -
From: Felipe Pina [EMAIL PROTECTED]
To: [EMAIL PROTECTED]
Sent: Wednesday, October 01, 2003 11:32 PM
Subject: [obm-l] Sequencias de Cauchy



 Gostaria que alguém esclarecesse a segunite dúvida.

 Seja (X,d) um espaço métrico e x_n uma seqüência satisfazendo
 d( x_(n+1), x_n ) - 0.
 Sejam m e n inteiros positivos diferentes... spg, m  n

 - x_m - x_n = x_m - x_(m-1) + x_(m-1) - x_(m-2) + x_(m-2) - 
 + x_(n+1) - x(n)

 Usando a desigualdade triangular...

 - 0 = d( x_m, x_n ) = d( x_m, x_(m-1)) + d( x_(m-1), x_(m-2)) + 
 + d( x_(n+1) , x(n) )

 Por que não posso concluir que x_n é Cauchy se cada termo do lado
 direito fica arbitrariamente pequeno ? Se fosse o caso da implicação ser
 verdadeira, teríamos que a série harmônica seria convergente, mas não
 estou conseguindo entender onde está a falha no raciocínio...

 Obrigado,
 Felipe Pina
 =
 Instruções para entrar na lista, sair da lista e usar a lista em
 http://www.mat.puc-rio.br/~nicolau/olimp/obm-l.html
 =


=
Instruções para entrar na lista, sair da lista e usar a lista em
http://www.mat.puc-rio.br/~nicolau/olimp/obm-l.html
=


[obm-l] Re: [obm-l] Equação

2003-09-30 Por tôpico Marcio Afonso A. Cohen
XY - X - Y = 0 =
XY - X - Y + 1 = 1 =
(Y-1)(X-1)=1 = Y-1 = -1 ou Y-1 = 1.

- Original Message -
From: Carlos [EMAIL PROTECTED]
To: [EMAIL PROTECTED]
Sent: Tuesday, September 30, 2003 8:32 PM
Subject: [obm-l] Equação


 Um aluno me passou uma equação de 1. Grau com duas
 incôgnitas.

 Quais os numeros inteiros que atendem a equação abaixo:

 XY = X + Y

 Por exemplo (0,0) (2,2) atendem a equação.

 Teria como ter uma saída algébrica?

 Agradeço



 __
 Acabe com aquelas janelinhas que pulam na sua tela.
 AntiPop-up UOL - É grátis!
 http://antipopup.uol.com.br/


 =
 Instruções para entrar na lista, sair da lista e usar a lista em
 http://www.mat.puc-rio.br/~nicolau/olimp/obm-l.html
 =


=
Instruções para entrar na lista, sair da lista e usar a lista em
http://www.mat.puc-rio.br/~nicolau/olimp/obm-l.html
=


Re: [obm-l] Resultados da Ibero 2003

2003-09-21 Por tôpico Marcio Afonso A. Cohen
Meus parabéns ao Fábio, ao Alex, ao Morgado e aos demais participantes
da lisdta belo resultado! Muito legal o fato de dois alunos fecharem a
prova! Parabéns!
Abracos,
Marcio

PS: O banco da Ibero tambem precisa ficar em segredo durante um certo tempo?
Ou ele pode ser divulgado prontamente?

- Original Message -
From: Augusto Cesar de Oliveira Morgado [EMAIL PROTECTED]
To: [EMAIL PROTECTED]
Sent: Sunday, September 21, 2003 4:27 AM
Subject: Re: [obm-l] Resultados da Ibero 2003


 Não, a correçao foi muito bem feita e o Alex bobeou. Eh claro que,
matematicamente, nao ha diferença entre o 42 do Fabio e o 41 do Alex. Mas o
Alex bobeou. Trocou um sinal, o que eh extremamente natural, e na hora de
fazer a conta final, em que teria a oportunidade de perceber que havia um
erro de sinal e, portanto, teria a oportunidade de corrigi-lo, nao fez a
conta final e escreveu o que deveria aparecer no final (se nao tivesse
errado), cometendo, portanto, um segundo erro de sinal.
 Uma coisa importante desta olimpiada foi o ingresso de Porto Rico no grupo
dos paises dourados, ou seja, que ganharam alguma medalha de ouro em Iberos.
Considero isso uma das melhores coisas desta Ibero. A outra foi a bailarina
de vermelho.
 Agora, falando serio(?): a festa de encerramento teve dois casais dançando
(ah, a bailarina de vermelho) e, para surpresa total, dançando nao o tango e
sim salsa, merengue, rumba, etc. Dizem as mas linguas que isso eh para
convencer a todos que a Argentina eh um pais caribenho e, portanto, pode
disputar a Centro-americana.
 A Olimpiada foi muito bem organizada, as condiçoes de hospedagem foram
luxuosas, nuestros hermanos portenos la hicieran muy bien.
 Como gozar argentinos eh, para nos, um esporte nacional, viva a seleçao
brasileira de polo. Passamos uma semana com a televisao enchendo nosso saco
com a propaganda do polo: assista a partida Argentina x Brasil e conheça o
melhor polo do mundo, o polo argentino.
 Resultado do jogo: Brasil 9x7.
 O que vou dizer agora eh o meu ponto de vista pessoal e nao deve nem pode
de modo algum ser interpretado como uma posiçao da Comissao de Olimpiadas. O
ponto fraco da Olimpiada, como sempre, eh o Jurado (conjunto dos chefes de
delegaçao). Escolheu mal a prova: o problema 6 nao eh um problema olimpico
(ou seja, um problema que necessite de mais engenho do que de conhecimentos
especificos para sua resoluçao), facilitando a vida de paises que fizeram um
treinamento aprofundado, como Brasil e Argentina. O problema 1 tambem nao
era suficientemente facil para evitar a crueldade de levar um menino a uma
Olimpiada e fazer com que ele volte para casa carregando seis notas zero.
 O desempenho do Brasil foi muito, muito bom. Sempre havera alguem que
diga: como, se houve um 0 numa questao? Isso acontece. Premido pelo tempo,
fica-se nervoso e aih eh que nao sai nada. Poderiamos ter tido um desempenho
melhor no problema 4. O resto, ateh o 0, eh da vida.
 Havia um problema absolutamente lindo no banco, mas nao foi selecionado.
 Abraços a todos.
 Morgado

 Em Fri, 19 Sep 2003 09:14:01 -0300, Claudio Buffara
[EMAIL PROTECTED] disse:

  Parabens a todos, especialmente Fabio e Alex que gabaritaram a prova (o
  ponto que la banquita tirou do Alex na no. 2 deve ter sido soh pra nao
ter
  2 brasileiros com nota maxima - aposto que o Morgado quase bateu em
algum
  hermano porte~no por causa disso - sou bairrista, sim, e dai?...)
 
  Um abraco,
  Claudio.
 
  on 18.09.03 19:59, Fábio Dias Moreira at [EMAIL PROTECTED]
  wrote:
 
   Oi pessoal,
  
   Já saíram os resultados da Ibero 2003:
  


=
Instruções para entrar na lista, sair da lista e usar a lista em
http://www.mat.puc-rio.br/~nicolau/olimp/obm-l.html
=


Re: [obm-l] Juros....

2003-09-14 Por tôpico Marcio Afonso A. Cohen
   Bom, voce jamais deve usar uma frase como essa: Isso nao eh usado em
nenhum lugar do mundo. No seu caso especifico, a frase eh falsa. Isso é
usado sim, bastante até! Por exemplo, quando se esta analisando carteiras de
acoes, em que os juros podem ser reinvestidos praticamente automaticamente,
considera-se capitalisacao continua, e o dinheiro anda no tempo com um fator
de e^(it).. Inclusive eu tenho certeza que isso eh usado na pratica, como
voce pode comprovar lendo livros como Options and Derivatives, de John
Hull, livro que inspira muitos investidores do mercado.. Acredito que o
Claudio Buffara possa te dar maiores informacoes sobre isso.
Abracos,
Marcio

- Original Message -
From: Artur Costa Steiner [EMAIL PROTECTED]
To: [EMAIL PROTECTED]
Sent: Sunday, September 14, 2003 3:29 AM
Subject: RE: [obm-l] Juros


Qual o melhor investimento10,25% ao ano, com juros compostos
semestralmente ou 10,20% ao ano com juros compostos continuamenteUm cara
me perguntou isso hoje, não tenho certeza sobre o enunciado, mas ele me
disse que viu esse problema em um livro do ElonAlguém já ouviu falar??
Será esse o enunciado correto???
Um abraço,
Crom

Eu jah vi juros compostos continuamente. Eh um conceito teorico, pois isto
nao eh usado na pratica em nenhum lugar do mundo. Seja i a taxa nominal de
juros, ao ano. De acordo com a convencao usual, se os juros forem
capitalizados em n periodos dentro do ano, entao a taxa efetiva de cada um
dos n periodos em que dividimos o ano serah de i/n. Logo, se o investidor
aplicar o principal P no inicio do primeiro periodo, apos t anos ele tera o
montante Mn = P(1+i/n)^nt, pois seu principal tera sido capitalizado nt
vezes. Se n tende ao infinito, entao cada periodo de capitalizacao tende a
zero e nos aproximamos cada vez mais de uma capitalizacao continua. Da
formula anterior, temos que se n- oo entao Mn - M1 = P e^(it). Por outro
lado, se a capitalizacao for semestral, entao apos t anos o investidor terah
o montante de M2 = P(1+i/2)^(2t), de acordo com a convencao usual. Para
saber o que eh melhor nos casos da questao, plote as duas curvas em funcao
de t, a semestral para i= 0,1025 e a continua para i=0,1020, supondo, para
facilitar, P = 1. A curva semestral deverah ser mais alta no inicio,
invertendo-se a situacao apos um certo t*, o chamado break-even point. A
melhor opcao provavelmente dependerah do tempo para se retirar o capital
investido (dependendo das taxas, pode acontecer que continua fique sempre
acima).
Artur


=
Instruções para entrar na lista, sair da lista e usar a lista em
http://www.mat.puc-rio.br/~nicolau/olimp/obm-l.html
=


[obm-l] Re: [obm-l] QUESTÃO OBM-U

2003-09-13 Por tôpico Marcio Afonso A. Cohen
Foi 24 na primeira fase, e no ano retrasado foi 15.

- Original Message -
From: Eduardo Casagrande Stabel [EMAIL PROTECTED]
To: [EMAIL PROTECTED]
Sent: Saturday, September 13, 2003 9:34 PM
Subject: [obm-l] QUESTÃO OBM-U


 Olá!

 Não vou falar sobre questão alguma. O título é só para atrair aqueles que
 pretendem falar sobre as questões antes de segunda-feira na lista. Por
 favor,

 NÃO COMENTEM AS QUESTÕES NA LISTA ATÉ SEGUNDA-FEIRA.

 Minha dúvida é quanto à nota de corte da OBM-u do ano passado, quanto foi?

 Abraçao!
 Duda.

 =
 Instruções para entrar na lista, sair da lista e usar a lista em
 http://www.mat.puc-rio.br/~nicolau/olimp/obm-l.html
 =


=
Instruções para entrar na lista, sair da lista e usar a lista em
http://www.mat.puc-rio.br/~nicolau/olimp/obm-l.html
=


Re: [obm-l] Contagem

2003-09-11 Por tôpico Marcio Afonso A. Cohen



Seguem alguns comentarios rapidos sobre esse 
problema.. Eh provavel que eu tenha errado as contas (nao conferi e fiz meio 
rapido), mas desse jeito foi bom que a resposta ficou simpatica.. 
Chame de x(n) as palavras de n letras sem dois A's 
adjacentes. 
Quantas palavras x(n+2) existem? 
 Se a primeira letra for A, há 
duas opções para a segunda letra (B ou C) e a partir daí temos x(n) 
opções.
 Caso contrário, há duas opções 
para a primeira letra (B ou C) e a partir daí temos x(n+1) opções.
Logo, x(n+2) = 2x(n+1) + 2x(n) (*)
 Usar funções geratrizes em geral 
não é uma boa técnica para resolverequações lineares de coeficientes 
constantes pq nesse caso tem uma teoria mais prática, muito parecida com a que 
voce usa para resolver EDOs..
 Sem maiores explicacoes sobre a 
teoria (qq coisa, de uma lida na Eureka ou mande um email que eu dou mais 
detalhes):
  Solucoes da 
forma t^n: t^2- 2t- 2 = 0 = t =1 +- sqrt(3), logo x(n) = 
a(1+sqrt(3))^n + b(1-sqrt(3))^n eh solucao de (*) qq que sejam a,b.
 No nosso caso porém, x(1)=3, 
x(2)=8 (donde a recorrencia da x(0) = 1) e portanto a+b=1, (a+b) + (a-b)sqrt(3) 
= 3 e então 
a = (2+sqrt(3))/2sqrt(3) = (1+sqrt(3))^2/8sqrt(3), 
b = (-2+sqrt(3))/2sqrt(3) = -(1-sqrt(3))^2/8sqrt(3)
 Logo, x(n) = [(1+sqrt(3))^(n+2) 
- (1-sqrt(3))^(n+2)]/8sqrt(3)
 Mais legal ainda é que, como 
(1-sqrt(3))^(n+2) / 8sqrt(3) eh quase sempre muito pequeno, e x(n) eh inteiro, 
voce pode concluir que:
n par: x(n) =Piso 
{(1+sqrt(3))^(n+2)/8sqrt(3)}
n impar: x(n) =Teto 
{(1+sqrt(3))^(n+2)/8sqrt(3)}

  - Original Message - 
  From: 
  Domingos Jr. 
  
  To: [EMAIL PROTECTED] 
  Sent: Thursday, September 11, 2003 8:47 
  PM
  Subject: Re: [obm-l] Contagem
  
  seja f(n) := número de palavras de n letras do 
  alfabeto {A, B, C} sem dois ou maisA's consecutivos
  e g(n) :=conta todas aspalavras 
  contadas por f(n)que terminam em A.
  f(1) = 3, g(1) = 1
  f(n + 1) = 3f(n) - g(n)
   [a idéia: uma palavra de n+1 
  letras deve ser formada por uma palavra de n letras mais uma letra, essa letra 
  pode ser A, B, C e a palavra anterior a ela não pode ter A's consecutivos, no 
  entanto se a palavra de tamanho n termina em A, não podemos colocar A como 
  última letra, logo descontamos g(n)]
  g(n + 1) = f(n) - g(n)
   [pegamos uma palavra de n 
  letras sem A's consecutivos eque NÃO termina em A e concatenamos um 
  A]
  
  
  agora vamos tentar resolver essas 
  recorrências!
  f(n) = g(n+1) + g(n) =
  f(n + 1) = g(n + 2) + g(n + 1)
  mas
  f(n + 1) = 3f(n) - g(n) = 3g(n + 1) + 
  2g(n)
  logo
  
  3g(n + 1) + 2g(n) = g(n + 2) + g(n + 
  1)
  g(n + 2) = 2[g(n + 1) + g(n)] = 
2f(n)
  f(n +2) = g(n + 3) + g(n + 2) = 2f(n + 1) + 
  2f(n)= 2[f(n+1) + f(n)]
  
  
  a recorrência passa a ser:
  f(1) = 3, f(2) = 8
  f(n +2)= 2[f(n+1) + f(n)], n = 
  1
  
  os primeiros valores são 3, 8, 22, 60, 164, 
  ...
  
  vamos obter a função geradora dessa nossa 
  f.
  seja A(x) = soma{i=1..oo} f(i)*x^i
  f(n +2)= 2[f(n+1) + f(n)] 
  =
  soma{i=1..oo} f(n + 2) = soma{i=1..oo} 2[f(n+1) + 
  f(n)] 
  
  temos:
  soma{i=1..oo} f(n + 2) = f(3)x + f(4)x² + ... = 
  [A(x) - f(1)x - f(2)x²]/x² = [A(x) -3x - 8x²]/x²
  soma{i=1..oo} f(n + 1) = f(2)x + f(3)x² + ... = 
  [A(x) - f(1)x]/x = [A(x) - 3x]/x
  
  logo:
  [A(x) -3x - 8x²]/x² = 2{[A(x) - 3x]/x + 
  A(x)}
  A(x) -3x - 8x² = 2{xA(x) - 3x² + 
  x²A(x)}
  A(x) (2x² + 2x - 1) = (-2x² - 3x)
  A(x) = (-2x² - 3x)/(2x² + 2x - 1) = -1 - 
  (x+1)(2x² + 2x - 1)
  
  
  precisamos agora calcular o coeficiente de x^n na 
  série que define A(x), fazer isso é um pouco trabalhoso e é bem técnico... o 
  livro do Herbert Wilf, generatingfunctionology calcula o falor de fib(n), a 
  sequência de Fibonacci... 
  
  devemosexpandir (x+1)/(1 - 2x - 2x²) em 
  "partial fractions" (não sei uma boa tradução).
  
  infelizmente estou apanhando pra fazer essa 
  expansão, fico te devendo!
  
  um lugar legal pra ver que vc acertou o problema 
  é:
  http://www.research.att.com/~njas/sequences/
  que tem um banco de dados grande de seqüências 
  inteiras, procure a sequência 3, 8, 22, 60, 164 pra vc ver que 
  legal!
  
  [ ]'s
  
  
- Original Message - 
From: 
[EMAIL PROTECTED] 
To: [EMAIL PROTECTED] 
Sent: Thursday, September 11, 2003 6:05 
PM
Subject: [obm-l] Contagem
Usando as letras A, B e C podemos formar 3^n "palavras" 
de n letras. Quantas dessas palavras não possuem dois ou mais A´s 
adjacentes??Esse exercício foi extraído do livro Problem-solving 
strategies, de Arthur Engel. Gostaria de ver outra solução, pois, a 
expressão final da minha solução está muito estranha...risos...eu diria 
...desengonçada. Se alguém fizer eu 
agradeço. 
Korshinoi 


Re: [obm-l] Conjunto denso em R

2003-09-09 Por tôpico Marcio Afonso A. Cohen
Espero que esteja certo, de uma conferida..

Se a eh irracional positivo, olhe para as aproximacoes por fracoes
continuas de a.
Temos a = a0 + 1/[a1 + 1/[a2+ e as reduzidas p_n/q_n (p0/q0 = a0,
p1/q1= a0+1/a1, p2/q2=a0+1/[a1+1/a2]... )
com n par satisfazem 0  a - p_n/q_n  (1/q_n)^2
Como os p_n,q_n sao positivos e tendem para infinito, podemos, dado um
eps0 qualquer, escolher n tq 1/q_n  eps.
Nesse caso, a desigualdade acima implica 0  (q_n)*a - p_n  eps.
Portanto, dado qualquer intervalo (r,r+eps) de R+, sempre existe algum
multiplo de (q_n)*a - p_n que cai nesse intervalo.
Para intervalos em R-, voce pode adotar uma ideia parecida, mas agora
olhando para as reduzidas de ordem impar.

Obs: As demonstracoes desses resultados sobre as reduzidas decorrem das
relacoes t(n+2) = a(n+2)t(n+1)+t(n), satisfeitas tanto por t(n)=p(n) quanto
por t(n)=q(n). Isso pode ser verificado por inducao, e pode ser conjecturado
a partir de uma analise das fracoes continuas de numeros racionais (que eh
o algoritmo de euclides).

Obs2: Se a = p/q, p,q inteiros, entao os elementos de B sao da forma
(np-mq)/q, e como o numerador eh inteiro, todos os elementos de B tem modulo
= 1/q. Em particular, B nao eh denso em R.
Se a for negativo, entao B soh tem elementos negativos e nao eh denso em
R.


- Original Message -
From: Claudio Buffara [EMAIL PROTECTED]
To: Lista OBM [EMAIL PROTECTED]
Sent: Tuesday, September 09, 2003 2:08 PM
Subject: [obm-l] Conjunto denso em R


 Um resultado relacionado que eu nao estou conseguindo provar (ou dar algum
 contra-exemplo) eh que o conjunto B = {n*a - m; m, n inteiros POSITIVOS}
eh
 denso em R.

 Qualquer ajuda serah bem-vinda.

 Um abraco,
 Claudio.

 =
 Instruções para entrar na lista, sair da lista e usar a lista em
 http://www.mat.puc-rio.br/~nicolau/olimp/obm-l.html
 =


=
Instruções para entrar na lista, sair da lista e usar a lista em
http://www.mat.puc-rio.br/~nicolau/olimp/obm-l.html
=


Re: [obm-l] Para os cariocas (treinamento)

2003-08-29 Por tôpico Marcio Afonso A. Cohen
Apenas confirmando, seremos eu e o Rodrigo Villard juntos. A gente vai
levar uma lista de problemas, metade temática (estilo semana olimpica) e o
resto sortido. O horario eh 14hs.
Abracos,
Marcio

- Original Message -
From: Carlos Gustavo Tamm de Araujo Moreira [EMAIL PROTECTED]
To: [EMAIL PROTECTED]
Sent: Friday, August 29, 2003 1:34 AM
Subject: [obm-l] Para os cariocas (treinamento)


Caros colegas,
No segundo semestre havera' reunioes semanais de treinamento olimpico
no
 IMPA, alternadamente as segundas e as tercas feiras. Na proxima terca, 2
de
 setembro, a reuniao sera' comandada pelo Marcio Cohen. A reuniao seguinte
 sera' na segunda, 8 de setembro, comandada pelo Luciano.
Abracos,
 Gugu
 =
 Instruções para entrar na lista, sair da lista e usar a lista em
 http://www.mat.puc-rio.br/~nicolau/olimp/obm-l.html
 =


=
Instruções para entrar na lista, sair da lista e usar a lista em
http://www.mat.puc-rio.br/~nicolau/olimp/obm-l.html
=


[obm-l] Re: [obm-l] V ou F Analítico.

2003-08-14 Por tôpico Marcio Afonso A. Cohen
Seja eps  0 dado.
Existe N tq nN implica |a(n) - a|  eps.
Seja A = |a(1)+a(2)+...+a(N) - Na|
Agora, fixando N, eps, A, temos que para todo natural n  N:
0=(1/n)*|a(1) + a(2) + ... + a(n) - na| = [A +
|a(N+1)-a|+|a(N+2)-a|+...+|a(n)-a|]/n  [A + (n-N)eps]/n
Tomando o limite quando n-oo dos dois lados da desigualdade acima (mantendo
A, N, eps fixos), obtemos:
0= lim (n-oo) de |( a(1)+a(2)+...+a(n) )/n - a| = eps
Como eps eh arbitrario (0), o limite acima deve valer zero e portanto sua
afirmativa eh verdadeira.


- Original Message -
From: Frederico Reis Marques de Brito [EMAIL PROTECTED]
To: [EMAIL PROTECTED]
Sent: Sunday, August 10, 2003 3:04 PM
Subject: [obm-l] V ou F Analítico.


 Bom pessoal, é o seguinte.

 Seja  a_n , n e IN , uma sequência de  reais  e suponha que   a_n - a .
 Verdadeiro ou Falso:


 (a_1 +  a_2  +  ... + a_n ) / n -  a.

 Infelizmente não sei como indicar um somatório ...

 Abraços,

 Frederico.

 _
 MSN Messenger: converse com os seus amigos online.
 http://messenger.msn.com.br

 =
 Instruções para entrar na lista, sair da lista e usar a lista em
 http://www.mat.puc-rio.br/~nicolau/olimp/obm-l.html
 =


=
Instruções para entrar na lista, sair da lista e usar a lista em
http://www.mat.puc-rio.br/~nicolau/olimp/obm-l.html
=


[obm-l] Neve caindo

2003-08-14 Por tôpico Marcio Afonso A. Cohen
Esse problema ocupou minha cabeca durante boa parte de uma palestra
chata que eu tive que assitir hoje. Consegui fazer no finalzinho da
palestra. Eh um bom treino para o pessoal que vai fazer a 2a fase da obm
agora (em particular aos que vao fazer a 1a fase da obm-u).
Numa determinada manha, numa determinada rua, comeca a cair neve
numa taxa constante. Ao meio dia, sai do quilometro zero dessa estrada um
carrinho limpador de neve, que tem a propriedade de retirar a neve numa
razao constante. Sabendo-se que ele passou pelo quilometro 2 as 14hs, e pelo
Km 3 as 16hs, determine o horario em que comecou a nevar.

Obs: Estou assumindo que a neve cai de forma igual independente da posicao
que voce se encontra na rua.

Abracos,
Marcio

=
Instruções para entrar na lista, sair da lista e usar a lista em
http://www.mat.puc-rio.br/~nicolau/olimp/obm-l.html
=


Re: [obm-l] Determinante

2003-08-03 Por tôpico Marcio Afonso A. Cohen



 Desse jeito que esta escrito, 
basta fazer o que o Fabio disse.. Acredito que o determinante que voce procura 
tenha um "X" na ultima coluna, ao lado do -1. Nesse caso fica mais interessante. 
Uma sugestao legal eh voce fazer os casos pequenos, conjecturar a resposta e 
provar por inducao.. (1x1: a0, 2x2: a(0)*x+a(1), 3x3: a(0)*x^2 + a(1)*x + a(2) 
... e em geral, usando Laplace, tente mostrar que det(nxn)=x*det(n-1 x n-1) + 
a(n-1)).
 Marcio.

- Original Message - 

  From: 
  ~*Åline*~ 
  To: [EMAIL PROTECTED] 
  Sent: Sunday, August 03, 2003 7:56 
  PM
  Subject: [obm-l] Determinante
  
  Alguém poderia me ajudar a calcular este 
  determinante?!
  
  
   
  a0 a1 a2 ... an-1an
  -1 X 0 
  ... 
  00
  0-1 X 
  ...0 
  0
  0 
  0-1 
  ...0 
  0
  
  0 0 
  0...X 
  0
  00 
  0 ... -1 
  0
  
  Aline


[obm-l] IMC na tv

2003-08-01 Por tôpico Marcio Afonso A. Cohen



 Espero que isso nao seja 
interpretado comomuito off-topic.
Eh bastante provavel que 
saia hoje, no Jornal Nacional, uma reportagem (ou talvez uma simples menção) a 
prova da olimpiada internacional de matemática universitaria (IMC) que ocorreu 
agora na Romenia, e o resultado do Brasil na mesma. 
 Nao eh garantido que va sair 
hoje, mas eh provavel..
 Abracos, 
 
Marcio


[obm-l] Re: [obm-l] Ajuda - Cálculo

2003-07-23 Por tôpico Marcio Afonso A. Cohen



 Se nao me engano, essa questao 
caiu na prova de admissao para a Escola Navalde 1998/1999, quando eu era 
vestibulando. Nenhuma das opcoes correspondia ao valor correto, e por isso a 
questao foi oficialmente anulada.


  - Original Message - 
  From: 
  Webmaster - Centrodador 
  To: [EMAIL PROTECTED] 
  Sent: Wednesday, July 23, 2003 12:02 
  AM
  Subject: [obm-l] Ajuda - Cálculo
  
  Caros companheiros, não estou conseguindo 
  resolver o sequinte problema:
  A equação da posição de um móvel num instante 
  t(em s)é dadapor: x(t)= cos^2(e^(t - pi/4)).sent(t) + cot^2(t), 
  com t maior ouigual a zero. O valor da aceleração do móvel no instante 
  t=0s é?
  As opções apresentam somente raizes de números 
  inteiros.
  []´s
  Igor Castro


Re: [obm-l] Re: [obm-l] IMO - P1

2003-07-19 Por tôpico Marcio Afonso A. Cohen
É verdade! Valeu!
Marcio

- Original Message -
From: [EMAIL PROTECTED]
To: [EMAIL PROTECTED]
Sent: Saturday, July 19, 2003 4:49 PM
Subject: [obm-l] Re: [obm-l] IMO - P1



 Oi Marcio,
  Soh hj eu li seu email, depois que eu tbm consegui fazer a questão.
  Tem apenas um detalhe que vc não observou: os t_i´s devem ser distintos,
 pq senão os dois conjuntos seriam iguais.
  Seguindo a sua notação, sendo D_i=(D+ t_i)U(t_i- D), temos |D_i|=
2.5050.
 O t_(i+1) deve ser escolhido em
T = S\(S_1 U...U S_i U {t_1, t_2,...,t_i})
   Olha como o problema é impressionante: para garantir que t_100 pode ser
 escolhido, devemos ter T não-vazio. Ora,
 |S_1 U...U S_99 U {t_1, t_2,...,t_99}|= |S_1|+...+ |S_99|+ 99=
   99.2.5050+ 99= 00+ 99= 99  100  ()
Os números foram muitos bem escolhidos, e o problema ainda não perdeu
 a elegância com números feios! NOvamente, parabéns Gugu.
   Ateh mais,

=
Instruções para entrar na lista, sair da lista e usar a lista em
http://www.mat.puc-rio.br/~nicolau/olimp/obm-l.html
=


Re: [obm-l] Problemas da IMO

2003-07-15 Por tôpico Marcio Afonso A. Cohen
Eu sei que ninguem gosta muito disso, mas esse problema 4 (que eu ateh
imagino que nao seja dificil por plana) eh bem simples na conta bruta.. Eh
impressionante como complexos ajudam nos problemas de geometria da imo..
aquele artigo da eureka 6 eh realmente muito util!

Coloque o circuncentro na origem, e represente os vertices pelos
complexos a,b,c,d, todos de modulo 1u.m.
Reta ab: z+abz' = a+b
Reta perpendicular a ab passando por d: z-abz'=d-abd'
Logo, o ponto P eh 2p = [a+b+d-ab/d]
Portanto, 2q = [a+c+d-ac/d] e 2r = [b+c+d-bc/d].
Como p,q,r sao colineares (reta de simpson), e |p-q| = |q-r|:
p-q = q-r, ou seja: b-c + ac/d - ab/d = a-b +bc/d-ac/d
Arrumando: (b-c) - (a/d)(b-c) = (a-b) - (c/d)(a-b) sse (b-c)(d-a)=(a-b)(d-c)
Tirando modulo, isso significa que BC*AD = AB*DC. E isso fecha o problema.
De fato, sendo I o peh da bissetriz de ABC em AC, entao, AI/IC = AB/BC e vc
quer provar que I eh peh da bissetriz de ADC, i.e, que AI/IC=AD/DC (teorema
da bissetriz interna, ida e volta). Portanto, eh suficiente provar que AB*DC
= AD*BC.

Vou pensar nos outros agora, esse foi o que eu achei que seria mais
facil.. (ja pensei no 2 e no 1 um pouco tmb..)


- Original Message -
From: [EMAIL PROTECTED]
To: [EMAIL PROTECTED]
Cc: [EMAIL PROTECTED]; [EMAIL PROTECTED]
Sent: Monday, July 14, 2003 3:38 PM
Subject: [obm-l] Problemas da IMO




 Prova da IMO retirada do Site http://www.mathlinks.go.ro/

 O Problema 1 é nois que mandou...


 First Day - 44th IMO 2003 Japan

 1. Let A be a 101-element subset of the set S={1,2,3,...,100}. Prove
that
 there exist numbers t_1, t_2, ..., t_{100} in S such that the sets

 Aj = { x + tj | x is in A } for each j = 1, 2, ..., 100

 are pairwise disjoint.


 2. Find all pairs of positive integers (a,b) such that the number

 a^2 / ( 2ab^2-b^3+1) is also a positive integer.

 3. Given is a convex hexagon with the property that the segment connecting
the
 middle points of each pair of opposite sides in the hexagon is  sqrt(3) /
2
 times the sum of those sides' sum.

 Prove that the hexagon has all its angles equal to 120.


 Second Day - 44th IMO 2003 Japan

 4. Given is a cyclic quadrilateral ABCD and let P, Q, R be feet of the
 altitudes from D to AB, BC and CA respectively. Prove that if PR = RQ then
the
 interior angle bisectors of the angles  ABC and  ADC are concurrent on
AC.

 5. Let x1 = x2 = ... = xn be real numbers, n2.

 a) Prove the following inequality:

 (sum  ni,j=1 | xi - xj | ) 2 = 2/3 ( n^2 - 1 )sum ni,j=1 ( xi - xj)^2

 b) Prove that the equality in the inequality above is obtained if and only
if
 the sequence (xk) is an arithemetical progression.

 6. Prove that for each given prime p there exists a prime q such that
n^p - p
 is not divisible by q for each positive integer n.



=
Instruções para entrar na lista, sair da lista e usar a lista em
http://www.mat.puc-rio.br/~nicolau/olimp/obm-l.html
=


Re: [obm-l] IMO - P1

2003-07-15 Por tôpico Marcio Afonso A. Cohen
Acho que consegui fazer  o 1o. Confiram ai e vejam se tem algum furo. O
2o eu realmente nao estou conseguindo.. Estou com alguma esperanca de fazer
o 5.. (o 3 eu tentei tmb, mas minhas contas estao muito grandes). Mandem
seus comentarios sobre a prova!
P1:
Note que (Ai inter Aj) != vazio sse existirem m,n tais que a_m + t_i =
a_n + t_j , i.e, a_m - a_n = t_j - t_i.
Vamos construir os t's indutivamente garantindo que isso nao acontece.
Existem binomial (101,2) = 5050 diferencas possiveis no conjunto A. Chame de
D={D1,D2,...D5050} o conjunto dessas diferencas (claro que algumas delas
podem ser iguais, mas temos |D| = 5050).
1. Escolha um t1 qualquer de S.
2. Agora quero garantir que t2-t1 e t1-t2 nao estao em D. Para isso, basta
escolher um elemento de S que nao esteja em
X1 = {t1+D1,t1+D2,...,t1+D5050}U{t1-D1, t1-D2,...,t1-D5050}. (pq se t2-t1
esta em D, entao t2=t1+Dk para algum k).
Isso eh facil pq |X1|=2.5050  |S|.

3. Agora vou escolher t3 em S garantindo que t3-t1, t1-t3, t3-t2, t2-t3 nao
estao em D.
Para isso, t3 nao pode estar em X1 e tmb nao pode estar em
X2 = {t2+D1,t2+D2,...,t2+D5050}U{t2-D1, t2-D2,...,t2-D5050}.
Isso eh facil, pq |X1 U X2| = 4.5050  |S|

Em geral, depois de escolhidos t1,t2,...,t_k-1, vou escolher t_k em S de
modo que ele nao esteja em nenhum dos conjuntos X1,X2,...,X_(k-1).
Para k=100, isso eh sempre possivel, pq |X1 U X2 U ... U X_(k-1)| =
2*(k-1)*5050 = 2*99*5050 = 00  10^6 = |S|.
(obs: X_s = {ts + D}U{ts-D}, na notacao usuao de x+A onde x eh um elemento e
A um conjunto).


Pronto. Foram escolhidos 100 t's tal que nao existe uma quadrupla (m,n,i,j)
tq a_m - a_n = t_j - t_i. (pois t_j - t_i esta sempre fora de D), e portanto
nunca se tem a_m + t_i = a_n + t_j, ou seja, as intersecoes sao todas vazias
de fato.

Abracos.



- Original Message -
From: [EMAIL PROTECTED]
To: [EMAIL PROTECTED]
Cc: [EMAIL PROTECTED]; [EMAIL PROTECTED]
Sent: Monday, July 14, 2003 3:38 PM
Subject: [obm-l] Problemas da IMO




 Prova da IMO retirada do Site http://www.mathlinks.go.ro/

 O Problema 1 é nois que mandou...


 First Day - 44th IMO 2003 Japan

 1. Let A be a 101-element subset of the set S={1,2,3,...,100}. Prove
that
 there exist numbers t_1, t_2, ..., t_{100} in S such that the sets

 Aj = { x + tj | x is in A } for each j = 1, 2, ..., 100

 are pairwise disjoint.


 2. Find all pairs of positive integers (a,b) such that the number

 a^2 / ( 2ab^2-b^3+1) is also a positive integer.

 3. Given is a convex hexagon with the property that the segment connecting
the
 middle points of each pair of opposite sides in the hexagon is  sqrt(3) /
2
 times the sum of those sides' sum.

 Prove that the hexagon has all its angles equal to 120.


 Second Day - 44th IMO 2003 Japan

 4. Given is a cyclic quadrilateral ABCD and let P, Q, R be feet of the
 altitudes from D to AB, BC and CA respectively. Prove that if PR = RQ then
the
 interior angle bisectors of the angles  ABC and  ADC are concurrent on
AC.

 5. Let x1 = x2 = ... = xn be real numbers, n2.

 a) Prove the following inequality:

 (sum  ni,j=1 | xi - xj | ) 2 = 2/3 ( n^2 - 1 )sum ni,j=1 ( xi - xj)^2

 b) Prove that the equality in the inequality above is obtained if and only
if
 the sequence (xk) is an arithemetical progression.

 6. Prove that for each given prime p there exists a prime q such that
n^p - p
 is not divisible by q for each positive integer n.



 -
 This mail sent through IMP: http://horde.org/imp/
 =
 Instruções para entrar na lista, sair da lista e usar a lista em
 http://www.mat.puc-rio.br/~nicolau/olimp/obm-l.html
 =


=
Instruções para entrar na lista, sair da lista e usar a lista em
http://www.mat.puc-rio.br/~nicolau/olimp/obm-l.html
=


Re: [obm-l] Problemas da IMO

2003-07-15 Por tôpico Marcio Afonso A. Cohen
Realmente, sua solucao me parece perfeita.. Alem de nao usar que o
quadrilatero eh inscritivel.. legal.
Voce pensou nos outros? Pensei bem no 2 e no 3, mas nao consegui fechar
nenhum.. O 3 eu acredito que seja alguma desigualdade virando igualdade, e
quero tentar mais pra ver se da certo..

- Original Message -
From: [EMAIL PROTECTED]
To: [EMAIL PROTECTED]
Sent: Tuesday, July 15, 2003 12:05 PM
Subject: Re: [obm-l] Problemas da IMO


 Marcio,
 achei legal essa sua solucao por complexos. Uma outra solucao
 trivial (e acho que a de 99% dos participantes) seria a seguinte:

 quad. APDR inscritivel  =  PR = AD.sen(BAC)
 quad. CQRD inscritivel  =  RQ = DC.sen(ACB)

 PR = RQ  =  AD/DC = sen(ACB)/sen(BAC) = AB/BC  (lei dos senos)   (*)

 Sendo S e T os pontos de interseccao das bissetrizes internas dos
 angulos ABC e ADC, respectivamente, com o lado AC, temos:

 AS/SC  =  AB/BC  =  AD/DC  =  AT/TC   Logo, S = T
   (1)   (2)   (3)

 (1) e (3) - teorema da bissetriz interna
 (2) - por (*)

 abracos,

 #
 # MSc. Edson Ricardo de A. Silva#
 # Computer Graphics Group (CRAB)#
 # Federal University of Ceara (UFC) #
 #

 On Tue, 15 Jul 2003, Marcio Afonso A. Cohen wrote:

  Eu sei que ninguem gosta muito disso, mas esse problema 4 (que eu
ateh
  imagino que nao seja dificil por plana) eh bem simples na conta bruta..
Eh
  impressionante como complexos ajudam nos problemas de geometria da imo..
  aquele artigo da eureka 6 eh realmente muito util!
 
  Coloque o circuncentro na origem, e represente os vertices pelos
  complexos a,b,c,d, todos de modulo 1u.m.
  Reta ab: z+abz' = a+b
  Reta perpendicular a ab passando por d: z-abz'=d-abd'
  Logo, o ponto P eh 2p = [a+b+d-ab/d]
  Portanto, 2q = [a+c+d-ac/d] e 2r = [b+c+d-bc/d].
  Como p,q,r sao colineares (reta de simpson), e |p-q| = |q-r|:
  p-q = q-r, ou seja: b-c + ac/d - ab/d = a-b +bc/d-ac/d
  Arrumando: (b-c) - (a/d)(b-c) = (a-b) - (c/d)(a-b) sse
(b-c)(d-a)=(a-b)(d-c)
  Tirando modulo, isso significa que BC*AD = AB*DC. E isso fecha o
problema.
  De fato, sendo I o peh da bissetriz de ABC em AC, entao, AI/IC = AB/BC e
vc
  quer provar que I eh peh da bissetriz de ADC, i.e, que AI/IC=AD/DC
(teorema
  da bissetriz interna, ida e volta). Portanto, eh suficiente provar que
AB*DC
  = AD*BC.
 
  Vou pensar nos outros agora, esse foi o que eu achei que seria mais
  facil.. (ja pensei no 2 e no 1 um pouco tmb..)
 
 
  - Original Message -
  From: [EMAIL PROTECTED]
  To: [EMAIL PROTECTED]
  Cc: [EMAIL PROTECTED]; [EMAIL PROTECTED]
  Sent: Monday, July 14, 2003 3:38 PM
  Subject: [obm-l] Problemas da IMO
 
 
  
  
   Prova da IMO retirada do Site http://www.mathlinks.go.ro/
  
   O Problema 1 é nois que mandou...
  
  
   First Day - 44th IMO 2003 Japan
  
   1. Let A be a 101-element subset of the set S={1,2,3,...,100}.
Prove
  that
   there exist numbers t_1, t_2, ..., t_{100} in S such that the sets
  
   Aj = { x + tj | x is in A } for each j = 1, 2, ..., 100
  
   are pairwise disjoint.
  
  
   2. Find all pairs of positive integers (a,b) such that the number
  
   a^2 / ( 2ab^2-b^3+1) is also a positive integer.
  
   3. Given is a convex hexagon with the property that the segment
connecting
  the
   middle points of each pair of opposite sides in the hexagon is
sqrt(3) /
  2
   times the sum of those sides' sum.
  
   Prove that the hexagon has all its angles equal to 120.
  
  
   Second Day - 44th IMO 2003 Japan
  
   4. Given is a cyclic quadrilateral ABCD and let P, Q, R be feet of the
   altitudes from D to AB, BC and CA respectively. Prove that if PR = RQ
then
  the
   interior angle bisectors of the angles  ABC and  ADC are concurrent
on
  AC.
  
   5. Let x1 = x2 = ... = xn be real numbers, n2.
  
   a) Prove the following inequality:
  
   (sum  ni,j=1 | xi - xj | ) 2 = 2/3 ( n^2 - 1 )sum ni,j=1 ( xi - xj)^2
  
   b) Prove that the equality in the inequality above is obtained if and
only
  if
   the sequence (xk) is an arithemetical progression.
  
   6. Prove that for each given prime p there exists a prime q such that
  n^p - p
   is not divisible by q for each positive integer n.
  
  
 
 
=
  Instruções para entrar na lista, sair da lista e usar a lista em
  http://www.mat.puc-rio.br/~nicolau/olimp/obm-l.html
 
=
 
 =
 Instruções para entrar na lista, sair da lista e usar a lista em
 http://www.mat.puc-rio.br/~nicolau/olimp/obm-l.html
 =


=
Instruções para entrar na lista, sair da lista e usar a lista em
http://www.mat.puc-rio.br/~nicolau/olimp/obm-l.html
=


[obm-l] Fw: [imo-problems] IMO 2003 problems are ONLINE

2003-07-14 Por tôpico Marcio Afonso A. Cohen
Que triste! Depois de passar boa parte da madrugada catando os problemas
da imo2003, acabo de receber uma msg de outra lista dizendo que eles  estao
online.. Justamente na hora que eu preciso sair.. bom, fica como diversao
para o pessoal da lista.. eu vou ter que, infelizmente, esperar um pouco..
Abracos,
Marcio


- Original Message -
From: Andrei Ismail [EMAIL PROTECTED]
To: [EMAIL PROTECTED]
Sent: Monday, July 14, 2003 9:40 AM
Subject: [imo-problems] IMO 2003 problems are ONLINE


 You can find them at the website :

 http://www.mathlinks.go.ro , at the forum
 section!

 Lots of other good stuff on that website.

 Cheers and happy solving everyone!

 Andrei

 P.S. : as far as I know this is the only place
 where they were displayed so far

 __
 Do you Yahoo!?
 SBC Yahoo! DSL - Now only $29.95 per month!
 http://sbc.yahoo.com

  Yahoo! Groups Sponsor -~--
 Buy 1, Get 1 FREE
 Control Cravings  Hunger EZ! Fast Acting Natural Oral Spray - $19.97
 http://www.challengerone.com/t/l.asp?cid=2866lp=ezappetite3.html
 http://us.click.yahoo.com/rJIe0D/99VGAA/ySSFAA/wHYolB/TM
 -~-

 imo-problems is part of the IMO network, http://imonet.online.fr/
 remember to go there and add your name !
 
 To receive these messages in a daily digest, email
[EMAIL PROTECTED]
 To stop recieving these messages, email [EMAIL PROTECTED]
 You can always view and reply to messages on-line, at
http://www.egroups.com/group/imo-problems
 For more help, email [EMAIL PROTECTED]


 Your use of Yahoo! Groups is subject to http://docs.yahoo.com/info/terms/




=
Instruções para entrar na lista, sair da lista e usar a lista em
http://www.mat.puc-rio.br/~nicolau/olimp/obm-l.html
=


[obm-l] Combinatoria

2003-07-11 Por tôpico Marcio Afonso A. Cohen



 Nao estou conseguindo fazer a 
seguinte questao, do livro de combinatoria do Morgado:
Um enxadrista joga partidas de xadrez durante onze 
semanas consecutivas. Sabe-se que ele sempre joga ao menos uma partida por dia, 
e jamais joga mais de 12 partidas em uma semana. Mostre que existe um periodo de 
dias consecutivos no qual ele joga exatamente 20 partidas.
 Alguem tem alguma 
dica?

 Abracos,
 
Marcio


[obm-l] Re: [obm-l] Como os Matemticos Complicam II

2003-07-08 Por tôpico Marcio Afonso A. Cohen



A questao eh perfeitamente coerente. Agora nao 
eh mais matematica.. Eh o minimo de logica.O fato ehque nao deu para 
VOCE entender o que a questao pede. Isso nao significa que a questao seja sem 
coesao. 
14.625 = 117/8 = Inteiro / n. 
Logo, n = Inteiro * 8 /117.
Como mdc(8,117) = 1, o menor n (lembre que n eh 
natural) ocorre quando Inteiro = 117, i.e, n = 8 (n=7,6,...,1 levam a Inteiro = 
Nao Inteiro).
Bastante interessante a questao.


  - Original Message - 
  From: 
  J.Paulo 
  roxer til the end 
  To: [EMAIL PROTECTED] 
  Sent: Tuesday, July 08, 2003 6:04 
PM
  Subject: [obm-l] Como os Matemticos 
  Complicam II
  
  
  
  "A mdia aritmtica dos elementos de um conjunto de nmeros 
  inteiros positivos  14,625. Se n  o nmero de elementos deste conjunto ento 
  o menor valor possvel de n :
  
  A)4
  B)6
  C)8
  D)16
  Mdia=soma/r
  14,625=s/r
  S=117r/8"
  Uma questo sem coeso.No d pra entender o q 
  ela pede.
  
  Email.it, the professional e-mail, 
  gratis per te: clicca qui 
  Sponsor:Stai cercando l'amore? Potrebbe essere pi vicino a te di 
  quanto credi!Clicca 
  qui 


[obm-l] cosh(x)

2003-07-07 Por tôpico Marcio Afonso A. Cohen



 Deparei-me com essa questao 
enquanto estudava uma prova antiga de uma olimpiada (imc). Embora tenha a 
solucao em (imc-math.org), achei a questao bem interessante e resolvi coloca-la 
aqui para que voces pensem nela.. Eu nao consegui fazer quando tentei e 
acabeicedendo a tentacao de olhar a resposta.. Tentem..
 "Mostre que se cosh(kx) e cosh 
(kx+x) sao racionais, entao cosh(x) tambem eh racional."
 
Abracos